You are on page 1of 67

First Aid Cases For The USMLE Step 1

4th Edition Edition Tao Le


Visit to download the full and correct content document:
https://ebookmass.com/product/first-aid-cases-for-the-usmle-step-1-4th-edition-editio
n-tao-le/
FIRST AID ®

CASES
FOR THE
USMLE
STEP 1
Fourth Edition

Tao Le, MD, MHS


Associate Clinical Professor
Chief, Section of Allergy and Immunology
Department of Medicine
University of Louisville School of Medicine

Richard A. Giovane, MD
Resident Physician
Department of Family Medicine
University of Alabama College of Community Health Sciences

New York / Chicago / San Francisco / Athens / London / Madrid / Mexico City
Milan / New Delhi / Singapore / Sydney / Toronto

00_USMLE-STEP1_00fm_i-xxiv.indd 1 9/17/18 2:27 PM


Copyright © 2019 by McGraw-Hill Education. All rights reserved. Except as permitted under the United States Copyright Act of 1976,
no part of this publication may be reproduced or distributed in any form or by any means, or stored in a database or retrieval system,
without the prior written permission of the publisher.

ISBN: 978-1-26-014314-0
MHID: 1-26-014314-7

The material in this eBook also appears in the print version of this title: ISBN: 978-1-26-014313-3,
MHID: 1-26-014313-9.

eBook conversion by codeMantra


Version 1.0

All trademarks are trademarks of their respective owners. Rather than put a trademark symbol after every occurrence of a trademarked
name, we use names in an editorial fashion only, and to the benefit of the trademark owner, with no intention of infringement of the
trademark. Where such designations appear in this book, they have been printed with initial caps.

McGraw-Hill Education eBooks are available at special quantity discounts to use as premiums and sales promotions or for use in corpo-
rate training programs. To contact a representative, please visit the Contact Us page at www.mhprofessional.com.

Notice

Medicine is an ever-changing science. As new research and clinical experience broaden our knowledge, changes in treatment and drug
therapy are required. The authors and the publisher of this work have checked with sources believed to be reliable in their efforts to
provide information that is complete and generally in accord with the standards accepted at the time of publication. However, in view
of the possibility of human error or changes in medical sciences, neither the authors nor the publisher nor any other party who has been
involved in the preparation or publication of this work warrants that the information contained herein is in every respect accurate or com-
plete, and they disclaim all responsibility for any errors or omissions or for the results obtained from use of the information contained in
this work. Readers are encouraged to confirm the information contained herein with other sources. For example and in particular, readers
are advised to check the product information sheet included in the package of each drug they plan to administer to be certain that the
information contained in this work is accurate and that changes have not been made in the recommended dose or in the contraindications
for administration. This recommendation is of particular importance in connection with new or infrequently used drugs.

TERMS OF USE

This is a copyrighted work and McGraw-Hill Education and its licensors reserve all rights in and to the work. Use of this work is subject
to these terms. Except as permitted under the Copyright Act of 1976 and the right to store and retrieve one copy of the work, you may
not decompile, disassemble, reverse engineer, reproduce, modify, create derivative works based upon, transmit, distribute, disseminate,
sell, publish or sublicense the work or any part of it without McGraw-Hill Education’s prior consent. You may use the work for your
own noncommercial and personal use; any other use of the work is strictly prohibited. Your right to use the work may be terminated if
you fail to comply with these terms.

THE WORK IS PROVIDED “AS IS.” McGRAW-HILL EDUCATION AND ITS LICENSORS MAKE NO GUARANTEES OR WAR-
RANTIES AS TO THE ACCURACY, ADEQUACY OR COMPLETENESS OF OR RESULTS TO BE OBTAINED FROM USING
THE WORK, INCLUDING ANY INFORMATION THAT CAN BE ACCESSED THROUGH THE WORK VIA HYPERLINK OR
OTHERWISE, AND EXPRESSLY DISCLAIM ANY WARRANTY, EXPRESS OR IMPLIED, INCLUDING BUT NOT LIMITED
TO IMPLIED WARRANTIES OF MERCHANTABILITY OR FITNESS FOR A PARTICULAR PURPOSE. McGraw-Hill Education
and its licensors do not warrant or guarantee that the functions contained in the work will meet your requirements or that its opera-
tion will be uninterrupted or error free. Neither McGraw-Hill Education nor its licensors shall be liable to you or anyone else for any
inaccuracy, error or omission, regardless of cause, in the work or for any damages resulting therefrom. McGraw-Hill Education has no
responsibility for the content of any information accessed through the work. Under no circumstances shall McGraw-Hill Education and/
or its licensors be liable for any indirect, incidental, special, punitive, consequential or similar damages that result from the use of or
inability to use the work, even if any of them has been advised of the possibility of such damages. This limitation of liability shall apply
to any claim or cause whatsoever whether such claim or cause arises in contract, tort or otherwise.
Dedication

To Tai Le, who brought us immeasurable love and joy.

iii

00_USMLE-STEP1_00fm_i-xxiv.indd 3 9/17/18 2:27 PM


This page intentionally left blank

00_USMLE-STEP1_00fm_i-xxiv.indd 4 9/17/18 2:27 PM


CONTENTS

Contributing Authors........................................................................................................................xv
Faculty Reviewers.............................................................................................................................xvii
Preface.................................................................................................................................................xix
Acknowledgements..........................................................................................................................xxi
How to Contribute...........................................................................................................................xxiii

SECTION I GENERAL PRINCIPLES

Chapter 1, Biochemistry..........................................................................................1
Case 1: Alkaptonuria..........................................................................................................................2
Case 2: Ataxia-Telangiectasia.........................................................................................................3
Case 3: Cyanide Poisoning..............................................................................................................4
Case 4: Down Syndrome .................................................................................................................5
Case 5: Duchenne Muscular Dystrophy.....................................................................................6
Case 6: Ehlers-Danlos Syndrome..................................................................................................7
Case 7: Familial Hypercholesterolemia.......................................................................................8
Case 8: Fragile X Syndrome.......................................................................................................... 10
Case 9: Fructose Intolerance........................................................................................................ 11
Case 10: Homocystinuria.............................................................................................................. 12
Case 11: Hurler Syndrome............................................................................................................ 13
Case 12: Kartagener Syndrome.................................................................................................. 14
Case 13: Lesch-Nyhan Syndrome.............................................................................................. 15
Case 14: McArdle Disease............................................................................................................. 16
Case 15: Phenylketonuria............................................................................................................. 17
Case 16: Pyruvate Dehydrogenase Deficiency..................................................................... 18
Case 17: Tay-Sachs Disease.......................................................................................................... 19
Case 18: Vitamin B1 Deficiency................................................................................................... 20
Case 19: Wiskott-Aldrich Syndrome...........................................................................................21

Chapter 2, Immunology.........................................................................................23
Case 1: Contact Dermatitis........................................................................................................... 24
Case 2: DiGeorge Syndrome........................................................................................................ 25
Case 3: Guillain-Barre Syndrome................................................................................................ 26
Case 4: Hemolytic Uremic Syndrome....................................................................................... 27
Case 5: Immunoglobulin A Deficiency.................................................................................... 28
Case 6: Onchocerciasis.................................................................................................................. 29

00_USMLE-STEP1_00fm_i-xxiv.indd 5 9/17/18 2:27 PM


Case 7: Osteomyelitis..................................................................................................................... 30
Case 8: Pneumocystis jiroveci Pneumonia............................................................................... 31
Case 9: Primary Tuberculosis....................................................................................................... 32
Case 10: Pseudomonas aeruginosa Infection......................................................................... 33
Case 11: Schistosomiasis............................................................................................................... 34
Case 12: Transplant Reaction....................................................................................................... 35
Case 13: Toxic Shock Syndrome..................................................................................................36

Chapter 3, Microbiology........................................................................................39
Case 1: Acanthamoeba Infection................................................................................................ 40
Case 2: Actinomyces Versus Nocardia........................................................................................ 41
Case 3: Anaphylaxis........................................................................................................................ 42
Case 4: Anthrax................................................................................................................................. 43
Case 5: Ascariasis............................................................................................................................. 44
Case 6: Aspergillosis....................................................................................................................... 45
Case 7: Botulism............................................................................................................................... 46
Case 8: Candidiasis.......................................................................................................................... 47
Case 9: Chagas Disease.................................................................................................................. 48
Case 10: Cholera............................................................................................................................... 49
Case 11: Chronic Granulomatous Disease.............................................................................. 50
Case 12: Clostridium difficile Infection...................................................................................... 51
Case 13: Congenital Syphilis........................................................................................................ 52
Case 14: Creutzfeldt-Jakob Disease.......................................................................................... 53
Case 15: Cryptococcal Meningitis.............................................................................................. 54
Case 16: Cysticercosis..................................................................................................................... 55
Case 17: Cytomegalovirus Infection......................................................................................... 56
Case 18: Dengue Fever.................................................................................................................. 57
Case 19: Diphtheria......................................................................................................................... 58
Case 20: Elephantiasis.................................................................................................................... 59
Case 21: Giardiasis........................................................................................................................... 60
Case 22: Group B Streptococcus Infection in Infants........................................................... 61
Case 23: Hand, Foot, and Mouth Disease................................................................................ 62
Case 24: Herpes Simplex Virus Type 2 Infection................................................................... 63
Case 25: Hookworm Infection..................................................................................................... 64
Case 26: Influenza Virus Infection.............................................................................................. 65
Case 27: Kaposi Sarcoma.............................................................................................................. 66
Case 28: Legionnaires Disease.................................................................................................... 67
Case 29: Leprosy.............................................................................................................................. 68
Case 30: Listeria Meningitis.......................................................................................................... 69
Case 31: Liver Cysts: Echinococcus Infection.......................................................................... 70
Case 32: Lyme Disease................................................................................................................... 71
Case 33: Malaria............................................................................................................................... 72
Case 34: Measles.............................................................................................................................. 73

vi

00_USMLE-STEP1_00fm_i-xxiv.indd 6 9/17/18 2:27 PM


Case 35: Mononucleosis................................................................................................................ 74
Case 36: Mucormycosis................................................................................................................. 75
Case 37: Mumps............................................................................................................................... 76
Case 38: Neisseria gonorrhoeae and Septic Arthritis........................................................... 77
Case 39: Neisseria meningitidis Meningitis.............................................................................. 78
Case 40: Pinworm Infection......................................................................................................... 79
Case 41: Rabies................................................................................................................................. 80
Case 42: Ringworm......................................................................................................................... 81
Case 43: Rocky Mountain Spotted Fever................................................................................ 82
Case 44: Shingles............................................................................................................................. 83
Case 45: Strongyloidiasis.............................................................................................................. 84
Case 46: System Mycoses............................................................................................................. 85
Case 47: Toxoplasmosis................................................................................................................. 86
Case 48: Visceral Leishmaniasis.................................................................................................. 87
Case 49: Yellow Fever......................................................................................................................88

Chapter 4, Pharmacology......................................................................................89
Case 1: Acetaminophen Overdose............................................................................................ 90
Case 2: Agranulocytosis Secondary to Drug Toxicity.......................................................... 92
Case 3: Barbiturate Versus Benzodiazepine........................................................................... 93
Case 4: β-Adrenergic Second Messenger Systems............................................................. 94
Case 5: Drug-Induced Lupus and Liver Metabolism........................................................... 95
Case 6: Drug Development.......................................................................................................... 96
Case 7: Ethylene Glycol Poisoning............................................................................................. 97
Case 8: Lead Poisoning.................................................................................................................. 98
Case 9: Monoamine Oxidase Inhibitor..................................................................................... 99
Case 10: Neuromuscular Blocking Drugs..............................................................................100
Case 11: Organophosphates and Cholinergic Drugs.......................................................101
Case 12: Pharmacodynamics.....................................................................................................102
Case 13: Pharmacokinetics........................................................................................................ 103

Chapter 5, Public Health Sciences.......................................................................105


Case 1: Alcohol Withdrawal.......................................................................................................106
Case 2: Benzodiazepine Overdose..........................................................................................107
Case 3: Delirium.............................................................................................................................108
Case 4: Drug Toxidromes............................................................................................................109
Case 5: Eating Disorders..............................................................................................................110
Case 6: Ethical Principles.............................................................................................................112
Case 7: Evaluation of Diagnostic Tests...................................................................................113
Case 8: Management of End-of-Life Care.............................................................................114
Case 9: Medicare............................................................................................................................115
Case 10: Milestones of Development.....................................................................................116
Case 11: Narcolepsy......................................................................................................................117
Case 12: Operant Conditioning................................................................................................118

vii

00_USMLE-STEP1_00fm_i-xxiv.indd 7 9/17/18 2:27 PM


Case 13: Opioid Intoxication......................................................................................................119
Case 14: Physician Confidentiality and Domestic Violence............................................120
Case 15: Statistical Bias................................................................................................................ 122

Chapter 6, Cardiovascular....................................................................................123
Case 1: Abdominal Aortic Aneurysm.....................................................................................124
Case 2: Aortic Stenosis.................................................................................................................125
Case 3: Atherosclerosis/Acute Coronary Syndrome.........................................................126
Case 4: Atrial Fibrillation.............................................................................................................127
Case 5: Atrial Myxoma..................................................................................................................128
Case 6: Atrioventricular Block...................................................................................................129
Case 7: Cardiac Anatomy and Biomarkers............................................................................130
Case 8: Coarctation of the Aorta..............................................................................................131
Case 9: Congenital Rubella........................................................................................................132
Case 10: Deep Venous Thrombosis.........................................................................................133
Case 11: Dilated Cardiomyopathy...........................................................................................134
Case 12: Endocarditis...................................................................................................................135
Case 13: Granulomatosis with Polyangiitis (Wegener Granulomatosis)....................136
Case 14: Heart Failure...................................................................................................................137
Case 15: Hypertension.................................................................................................................138
Case 16: Hypertrophic Cardiomyopathy...............................................................................139
Case 17: Kawasaki Disease.........................................................................................................140
Case 18: Mesenteric Ischemia...................................................................................................141
Case 19: Mitral Valve Prolapse...................................................................................................142
Case 20: Myocardial Infarction..................................................................................................143
Case 21: Patent Ductus Arteriosus..........................................................................................144
Case 22: Pericarditis......................................................................................................................145
Case 23: Polyarteritis Nodosa....................................................................................................146
Case 24: Rheumatic Heart Disease..........................................................................................147
Case 25: Temporal Arteritis........................................................................................................148
Case 26: Tetralogy of Fallot.........................................................................................................149
Case 27: Truncus Arteriosus.......................................................................................................150
Case 28: Unstable Angina...........................................................................................................151
Case 29: Wolff-Parkinson-White Syndrome.......................................................................... 152

Chapter 7, Endocrine............................................................................................153
Case 1: Adrenal Insufficiency.....................................................................................................154
Case 2: Congenital Adrenal Hyperplasias.............................................................................156
Case 3: Cushing Syndrome and Glucocorticoid Excess...................................................158
Case 4: Graves Disease.................................................................................................................160
Case 5: Growth Hormone Excess, Gigantism, and Acromegaly....................................161
Case 6: Hyperparathyroidism....................................................................................................162
Case 7: Hypothyroidism..............................................................................................................163

viii

00_USMLE-STEP1_00fm_i-xxiv.indd 8 9/17/18 2:27 PM


Case 8: Metabolic Syndrome.....................................................................................................164
Case 9: Multiple Endocrine Neoplasia....................................................................................165
Case 10: Primary Hyperaldosteronism...................................................................................166
Case 11: Pseudohypoparathyroidism....................................................................................168
Case 12: Pheochromocytoma...................................................................................................170
Case 13: Sheehan Syndrome.....................................................................................................171
Case 14: Syndrome of Inappropriate Antidiuretic Hormone.........................................172
Case 15: Thyroglossal Duct Cyst...............................................................................................173
Case 16: Thyroid Cancer..............................................................................................................174
Case 17: Toxic Multinodular Goiter.........................................................................................175
Case 18: Type 1 Diabetes Mellitus and Diabetic Ketoacidosis.......................................176
Case 19: Type 2 Diabetes Mellitus...........................................................................................178
Case 20: Thyroidectomy.............................................................................................................. 180

Chapter 8, Gastrointestinal.................................................................................181
Case 1: Achalasia............................................................................................................................182
Case 2: Acute Pancreatitis...........................................................................................................184
Case 3: Alcoholic Cirrhosis of the Liver..................................................................................185
Case 4: Appendicitis.....................................................................................................................186
Case 5: Cholangitis........................................................................................................................188
Case 6: Crigler-Najjar Syndrome and Hereditary Hyperbilirubinemias.....................189
Case 7: Diverticulitis.....................................................................................................................190
Case 8: Esophageal Atresia with Tracheoesophageal Fistula........................................192
Case 9: Gallstones (Cholelithiasis)...........................................................................................194
Case 10: Gastric Carcinoma........................................................................................................195
Case 11: Gastrinoma.....................................................................................................................196
Case 12: Gastroesophageal Reflux Disease and Barrett Esophagus...........................197
Case 13: Hemochromatosis.......................................................................................................198
Case 14: Hepatitis B Virus Infection.........................................................................................199
Case 15: Hepatitis C Virus Infection.........................................................................................200
Case 16: Hepatocellular Carcinoma........................................................................................201
Case 17: Hyperbilirubinemia.....................................................................................................202
Case 18: Inflammatory Bowel Diseases.................................................................................203
Case 19: Intussusception and Meckel Diverticulum.........................................................204
Case 20: Lower Gastrointestinal Bleeding/Diverticulosis...............................................205
Case 21: Primary Biliary Cirrhosis.............................................................................................206
Case 22: Primary Hepatocellular Carcinoma........................................................................207
Case 23: Pseudomembranous Colitis and Clostridium difficile Infection...................208
Case 24: Pyloric Stenosis.............................................................................................................209
Case 25: Reye Syndrome.............................................................................................................210
Case 26: Short Bowel Syndrome..............................................................................................211
Case 27: Upper Gastrointestinal Bleed..................................................................................212
Case 28: Vitamin B12 Deficiency................................................................................................213

ix

00_USMLE-STEP1_00fm_i-xxiv.indd 9 9/17/18 2:27 PM


Case 29: Vitamin B3 Deficiency.................................................................................................214
Case 30: Zollinger-Ellison Syndrome...................................................................................... 215

Chapter 9, Hematology and Oncology...............................................................217


Case 1: Acute Intermittent Porphyria.....................................................................................218
Case 2: Acute Lymphoblastic Leukemia................................................................................219
Case 3: Acute Myelogenous Leukemia..................................................................................220
Case 4: Anatomy of Spleen........................................................................................................221
Case 5: Anemias: Aplastic Anemia...........................................................................................222
Case 6: Anemias: Iron Deficiency Anemia............................................................................223
Case 7: Anemias: Macrocytic Anemia.....................................................................................224
Case 8: Anemias: Sickle Cell Anemia.......................................................................................225
Case 9: Anemias: Warm Autoimmune Hemolytic Anemia..............................................226
Case 10: Breast Cancer.................................................................................................................227
Case 11: Burkitt Lymphoma and Tumor Lysis Syndrome................................................228
Case 12: Carcinoid Syndrome...................................................................................................229
Case 13: Chronic Myelogenous Leukemia............................................................................230
Case 14: Colorectal Cancer.........................................................................................................231
Case 15: Disseminated Intravascular Coagulation............................................................232
Case 16: Gastric Cancer...............................................................................................................234
Case 17: Glucose-6-Phospate Dehydrogenase Deficiency.............................................235
Case 18: Head and Neck Squamous Cell Tumor.................................................................236
Case 19: Hereditary Hemochromatosis.................................................................................237
Case 20: Hodgkin Lymphoma...................................................................................................238
Case 21: Idiopathic Thrombocytopenic Purpura...............................................................240
Case 22: Hereditary Spherocytosis..........................................................................................242
Case 23: Lead Poisoning.............................................................................................................243
Case 24: Lung Cancer...................................................................................................................244
Case 25: Multiple Myeloma........................................................................................................245
Case 26: Neuroblastoma.............................................................................................................246
Case 27: Oligodendroglioma....................................................................................................247
Case 28: Ovarian Cancer.............................................................................................................248
Case 29: Pancreatic Cancer........................................................................................................249
Case 30: Platelet and Coagulation Disorders and Hemophilia.....................................250
Case 31: Polycythemia Vera.......................................................................................................252
Case 32: Retinoblastoma............................................................................................................253
Case 33: Small Cell Lung Cancer..............................................................................................254
Case 34: Teratoma.........................................................................................................................255
Case 35: Testicular Cancer..........................................................................................................256
Case 36: Thalassemias..................................................................................................................257
Case 37: Thrombotic Thrombocytopenic Purpura and
Hemolytic-Uremic Syndrome..................................................................................258
Case 38: Von Willebrand Disease............................................................................................. 259

00_USMLE-STEP1_00fm_i-xxiv.indd 10 9/17/18 2:27 PM


Chapter 10, Musculoskeletal, Skin, and Connective Tissue..............................261
Case 1: Abdominal and Peritoneum Anatomy...................................................................262
Case 2: Anatomy of Thorax.........................................................................................................263
Case 3: Costochondritis...............................................................................................................264
Case 4: Ewing Sarcoma................................................................................................................265
Case 5: Gout.....................................................................................................................................266
Case 6: Hip Fracture......................................................................................................................267
Case 7: Inguinal Hernias..............................................................................................................268
Case 8: Klumpke Palsy..................................................................................................................269
Case 9: Knee Anatomy.................................................................................................................270
Case 10: Melanoma.......................................................................................................................272
Case 11: Muscular Dystrophies.................................................................................................273
Case 12: Osteoarthritis.................................................................................................................274
Case 13: Neurofibromatosis.......................................................................................................276
Case 14: Osteogenesis Imperfecta..........................................................................................277
Case 15: Osteoporosis..................................................................................................................278
Case 16: Rheumatoid Arthritis..................................................................................................279
Case 17: Rotator Cuff Tear...........................................................................................................280
Case 18: Squamous Cell Carcinoma........................................................................................281
Case 19: Systemic Lupus Erythematosus..............................................................................282
Case 20: Systemic Sclerosis........................................................................................................ 283

Chapter 11, Neurology and Special Senses........................................................285


Case 1: Alzheimer Disease..........................................................................................................286
Case 2: Amyotrophic Lateral Sclerosis...................................................................................287
Case 3: Acute Otitis Media..........................................................................................................288
Case 4: Brown-Séquard Syndrome..........................................................................................289
Case 5: Central Cord Syndrome................................................................................................290
Case 6: Corneal Abrasion and Eye Injury...............................................................................292
Case 7: Craniopharyngioma......................................................................................................293
Case 8: Femoral Neuropathy.....................................................................................................294
Case 9: Glioblastoma Multiforme............................................................................................295
Case 10: Guillain-Barré Syndrome...........................................................................................296
Case 11: Hearing Loss..................................................................................................................297
Case 12: Homonymous Hemianopia and Visual Field Defects......................................298
Case 13: Horner Syndrome........................................................................................................300
Case 14: Huntington Disease....................................................................................................301
Case 15: Hydrocephalus..............................................................................................................302
Case 16: Macular Degeneration...............................................................................................303
Case 17: Medulloblastoma.........................................................................................................304
Case 18: Meningioma...................................................................................................................305
Case 19: Metastatic Brain Tumor..............................................................................................306
Case 20: Migraine Headache.....................................................................................................307

xi

00_USMLE-STEP1_00fm_i-xxiv.indd 11 9/17/18 2:27 PM


Case 21: Multiple Sclerosis.........................................................................................................308
Case 22: Myasthenia Gravis........................................................................................................310
Case 23: Neurofibromatosis Type 1.........................................................................................311
Case 24: Neurofibromatosis Type 2 and Hearing Loss.....................................................312
Case 25: Open-Angle Glaucoma..............................................................................................313
Case 26: Parkinson Disease........................................................................................................314
Case 27: Pituitary Adenoma......................................................................................................316
Case 28: Recurrent Laryngeal Nerve Injury..........................................................................317
Case 29: Seizure..............................................................................................................................318
Case 30: Spinal Cord Compression due to Metastatic Cancer.......................................319
Case 31: Stroke...............................................................................................................................320
Case 32: Sturge-Weber Syndrome...........................................................................................321
Case 33: Subarachnoid Hemorrhage......................................................................................322
Case 34: Subdural Hematoma..................................................................................................323
Case 35: Transient Ischemic Attack.........................................................................................324
Case 36: Syncope...........................................................................................................................326
Case 37: Tuberous Sclerosis.......................................................................................................327
Case 38: Ulnar Neuropathy........................................................................................................328
Case 39: Vascular Dementia.......................................................................................................329
Case 40: Viral Meningitis.............................................................................................................330
Case 41: Von Hippel–Lindau Disease.....................................................................................331
Case 42: Wernicke Encephalopathy and Vitamin B1 Deficiency................................... 332

Chapter 12, Psychiatry.........................................................................................333


Case 1: Attention-Deficit Hyperactivity Disorder...............................................................334
Case 2: Bipolar Disorder..............................................................................................................335
Case 3: Generalized Anxiety Disorder....................................................................................336
Case 4: Major Depressive Disorder..........................................................................................337
Case 5: Obsessive-Compulsive Disorder...............................................................................338
Case 6: Obsessive-Compulsive Personality Disorder........................................................339
Case 7: Panic Disorder..................................................................................................................340
Case 8: Post-traumatic Stress Disorder..................................................................................341
Case 9: Schizophreniform Disorder.........................................................................................342
Case 10: Somatic Symptom Disorder.....................................................................................343
Case 11: Steroid-Induced Mania..............................................................................................344
Case 12: Tardive Dyskinesia.......................................................................................................345
Case 13: Tourette Syndrome...................................................................................................... 346

Chapter 13, Renal.................................................................................................347


Case 1: Acute Interstitial Nephritis..........................................................................................348
Case 2: Acute Pyelonephritis.....................................................................................................349
Case 3: Acute Tubular Necrosis.................................................................................................350
Case 4: Autosomal Dominant Polycystic Kidney Disease...............................................352
Case 5: Fanconi Syndrome.........................................................................................................354

xii

00_USMLE-STEP1_00fm_i-xxiv.indd 12 9/17/18 2:27 PM


Case 6: Goodpasture Syndrome...............................................................................................356
Case 7: Henoch-Schönlein Purpura........................................................................................358
Case 8: Hereditary Nephritis (Alport Syndrome)................................................................359
Case 9: Hypercalcemia.................................................................................................................360
Case 10: Hypokalemia..................................................................................................................362
Case 11: Hyponatremia...............................................................................................................363
Case 12: Hyponatremia...............................................................................................................364
Case 13: Hypophosphatemic Rickets.....................................................................................365
Case 14: Metabolic Acidosis with Respiratory Alkalosis..................................................366
Case 15: Metabolic Alkalosis......................................................................................................367
Case 16: Nephrolithiasis..............................................................................................................368
Case 17: Nephrotic Syndrome: Focal Segmental Glomeruloscleritis..........................369
Case 18: Nephrotic Syndrome: Minimal Change Disease...............................................370
Case 19: Rapidly Progressive Glomerulonephritis.............................................................371
Case 20: Renal Artery Stenosis..................................................................................................372
Case 21:Transplant Immunology.............................................................................................374
Case 22: Vesicoureteral Reflux.................................................................................................. 375

Chapter 14, Reproductive....................................................................................377


Case 1: Abruptio Placentae........................................................................................................378
Case 2: Androgen Insensitivity Syndrome............................................................................380
Case 3: Bacterial Vaginosis.........................................................................................................381
Case 4: Benign Prostatic Hyperplasia.....................................................................................382
Case 5: Breast Mass.......................................................................................................................383
Case 6: Ectopic Pregnancy.........................................................................................................384
Case 7: Endometriosis..................................................................................................................385
Case 8: Erectile Dysfunction......................................................................................................386
Case 9: Gestational Diabetes Mellitus....................................................................................387
Case 10: Hydatidiform Mole......................................................................................................388
Case 11: Klinefelter Syndrome..................................................................................................389
Case 12: Leiomyoma.....................................................................................................................390
Case 13: Menopause....................................................................................................................391
Case 14: Ovarian Cancer.............................................................................................................392
Case 15: Paget Disease of the Breast......................................................................................393
Case 16: Pelvic Inflammatory Disease....................................................................................394
Case 17: Preeclampsia..................................................................................................................395
Case 18: Testicular Torsion..........................................................................................................396
Case 19: Turner Syndrome.......................................................................................................... 397

Chapter 15, Respiratory.......................................................................................399


Case 1: Acute Epiglottitis............................................................................................................400
Case 2: Acute Respiratory Distress Syndrome.....................................................................401
Case 3: Anatomy of Lung............................................................................................................402
Case 4: Asthma Exacerbation....................................................................................................403

xiii

00_USMLE-STEP1_00fm_i-xxiv.indd 13 9/17/18 2:27 PM


Case 5: Bronchiectasis..................................................................................................................404
Case 6: Carbon Monoxide Poisoning.....................................................................................405
Case 7: Chronic Obstructive Pulmonary Disease...............................................................406
Case 8: Cystic Fibrosis...................................................................................................................407
Case 9: Emphysema......................................................................................................................408
Case 10: Neonatal Respiratory Distress Syndrome............................................................410
Case 11: Pleural Effusion.............................................................................................................411
Case 12: Pneumoconioses..........................................................................................................412
Case 13: Pneumonia: Community-Acquired Pneumonia...............................................414
Case 14: Pneumonia: Mycoplasma pneumoniae Infection..............................................415
Case 15: Pneumothorax..............................................................................................................416
Case 16: Respiratory Acidosis....................................................................................................417
Case 17: Sarcoidosis......................................................................................................................418
Case 18: Small Cell Lung Cancer..............................................................................................419
Case 19: Venous Thromboembolism...................................................................................... 420

Appendix: Case Index...........................................................................................423


Index .....................................................................................................................431
About the Editors.................................................................................................463

xiv

00_USMLE-STEP1_00fm_i-xxiv.indd 14 9/17/18 2:27 PM


CONTRIBUTING AUTHORS

Alyssa D. Brown Jonathan K. Lim, MD


University of Louisville Medical School Rutgers New Jersey Medical School
Class of 2019 Class of 2017

John E. Coda Varayini Pankayatselvan


Pennsylvania State University College of Medicine University of New Mexico School of Medicine
Class of 2019 Class of 2019

Eric L. Cox, DO Nand V. Patel, MD


Michigan State University College of Osteopathic Medicine Ross University School of Medicine
Class of 2018 Class of 2018

Jason Feinman, MD Andrew J. Regent-Smith, MD


Rutgers-Robert Wood Johnson Medical School Medical College of Wisconsin
Class of 2018 Class of 2018

Morris M. Kim Rahul Sheth


Tulane University School of Medicine University of Miami Miller School of Medicine
Class of 2019 Class of 2019

xv

00_USMLE-STEP1_00fm_i-xxiv.indd 15 9/17/18 2:27 PM


This page intentionally left blank

00_USMLE-STEP1_00fm_i-xxiv.indd 16 9/17/18 2:27 PM


FACULTY REVIEWERS

Sheldon Campbell, MD, PhD Jessica M. Jones, PhD


Professor of Laboratory Medicine Associate Professor, Department of Biochemistry and
Yale School of Medicine Molecular and Cellular Biology
Georgetown University School of Medicine
Brooks D. Cash, MD
Professor of Medicine, University of Texas McGovern Christopher C. Keller, PhD
Medical School Associate Professor of Microbiology and Immunology
Division Director, Gastroenterology, Hepatology, and Director of Microbiology, Immunology, and Pharmacology
Nutrition Lake Erie College of Osteopathic Medicine
Chief, Gastroenterology/Hepatology
Memorial Herman Hospital and Lyndon B. Johnson Fareeha Khalil, MD
General Hospital Assistant Professor of Medicine/Nephrology, Department
Co-Director, Ertan Digestive Disease Center of Excellence of Medicine, Nephrology
University of Texas Health Science Center at Houston Penn State Health, Milton S. Hershey Medical Center

Kenneth M. Certa, MD Suzanne N. King, PhD


Associate Professor, Department of Psychiatry and Assistant Professor, Department of Otolaryngology-Head
Human Behavior and Neck Surgery and Communicative Disorders
Sidney Kimmel Medical College at Thomas Jefferson University of Louisville School of Medicine
University
Psychiatry Residency Program Director Jack Kues, PhD
Thomas Jefferson University Hospital Professor Emeritus of Family and Community Medicine
Associate Dean for Continuous Professional Development
Jennifer Fung, MD University of Cincinnati College of Medicine
Assistant Professor, Division of Pulmonary and Critical Care
Medicine, Department of Medicine Gerald Lee, MD
Mount Sinai St Luke’s, Icahn School of Medicine at Assistant Professor, Department of Basic Sciences
Mount Sinai Emory University School of Medicine

Babak Ghavami, MD Kachiu Lee, MD, MPH


Clinical Teaching Fellow Assistant Clinical Professor, Department of Dermatology
Ross University School of Medicine The Warren Alpert Medical School of Brown University

William Jens, MD Kurt Ludwig, DO


Clinical Neurophysiology Fellow, Department of Neurology Program Director, Obstetrics and Gynecology Residency
Penn State Health, Milton S. Hershey Medical Center Program
Henry Ford Macomb Hospital
Catherine Jones, MD
Assistant Professor of Medicine, Department of Medicine Sam MacBride, MD
Associate Program Director, Internal Medicine Residency Family Practice Physician
Program Gallup, New Mexico
Tulane University School of Medicine
xvii

00_USMLE-STEP1_00fm_i-xxiv.indd 17 9/17/18 2:27 PM


Carl Marfurt, PhD Shirleen Madani Sims, MD
Professor of Anatomy and Cell Biology Associate Professor and Clerkship Director, Department of
Indiana University School of Medicine-Northwest-Gary Obstetrics and Gynecology
University of Florida College of Medicine
Brandon Mauldin, MD, MHS
Assistant Professor of Clinical Medicine, Department of David Sosa, MD
Internal Medicine Department of Internal Medicine
Tulane University School of Medicine University of New Mexico

Swapan K. Nath, PhD Gerald R. Thrush, PhD


Professor of Medical Education, School of Medicine Professor of Immunology
Texas Christian University and University of North Texas Associate Dean, Academic Affairs
Health Science Center College of Osteopathic Medicine of the Pacific
Western University of Health Sciences
Kelly M. Quesnelle, PhD
Assistant Professor, Department of Biomedical Sciences Ashish Verma, MD
Western Michigan University Homer Stryker MD School of Assistant Clinical Professor, Department of Medicine
Medicine Michigan State University College of Osteopathic Medicine

Vijay Rajput, MD
Professor and Chair, Department of Internal Medicine
Associate Dean, Academic and Student Affairs
Ross University School of Medicine

xviii

00_USMLE-STEP1_00fm_i-xxiv.indd 18 9/17/18 2:27 PM


PREFACE

With First Aid Cases for the USMLE Step 1, 4th edition, we continue our commitment
to providing students with the most useful and up-to-date preparation guides for
the USMLE Step 1 exam. This edition represents an outstanding effort by a talented
group of authors and includes the following:

• A new full-color design and full-color figures enhance clinical images and
illustrations and clarify text presentation.
• A new dedicated Immunology chapter.
• Updated USMLE-style cases with expanded differentials and commonly asked
question stems seen on the USMLE Step 1 exam.
• Many new high-yield figures and tables complement the questions and
answers.
• Concise yet complete with relevant pathophysiology explanations.
• Organized as a perfect supplement to First Aid for the USMLE Step 1.

We invite you to share your thoughts and ideas to help us improve First Aid Cases for
the USMLE Step 1. See “How to Contribute,” on p. xxiii.

Louisville Tao Le
Tuscaloosa Richard A. Giovane

xix

00_USMLE-STEP1_00fm_i-xxiv.indd 19 9/17/18 2:27 PM


This page intentionally left blank

00_USMLE-STEP1_00fm_i-xxiv.indd 20 9/17/18 2:27 PM


ACKNOWLEDGMENTS

We gratefully acknowledge the thoughtful comments, corrections, and advice of


the many medical students, international medical graduates, and faculty who have
supported the authors in the development of First Aid Cases for the USMLE Step 1,
4th edition.

For support and encouragement throughout the process, we are grateful to Thao
Pham. We thank Susan Mazik, Hans Neuhart, and Dr. Kimberly Kallianos for their
work on the new and updated images and illustrations. For administrative support
we thank Louise Petersen.

Thanks to our publisher, McGraw-Hill Education, for the valuable assistance of their
staff, Bob Boehringer and Christina Thomas. For exceptional editorial leadership,
enormous thanks to Emma Underdown and Isabel Nogueira. For outstanding edi-
torial work, we thank Linda Thomas. A special thanks to Graphic World and Kelsey
Smith for remarkable production work.

For contributions and corrections, we thank Dallas Chase, Daanish Chawala, Sean
Coeckelenbergh, Ryan Hadden, Andrew Hattel, Siva Iyer, and Nazmi Maher Al Qutub.

Louisville Tao Le
Tuscaloosa Richard A. Giovane

xxi

00_USMLE-STEP1_00fm_i-xxiv.indd 21 9/17/18 2:27 PM


This page intentionally left blank

00_USMLE-STEP1_00fm_i-xxiv.indd 22 9/17/18 2:27 PM


HOW TO CONTRIBUTE

To continue to produce a high-yield review source for the USMLE Step 1 you are
invited to submit any suggestions or corrections. We also offer paid internships in
medical education and publishing ranging from three months to one year (see be-
low for details). Please send us your suggestions for:

• New facts, mnemonics, diagrams, and clinical images


• High-yield topics that may reappear on future Step 1 exams
• Corrections and other suggestions

For each entry incorporated into the next edition, you will receive up to a $20
Amazon.com gift card as well as personal acknowledgment in the next edition. Sig-
nificant contributions will be compensated at the discretion of the authors. Also
let us know about material in this edition that you feel is low yield and should be
deleted.

All submissions including potential errata should ideally be supported with hyper-
links to a dynamically updated Web resource such as UpToDate, AccessMedicine, and
ClinicalKey.

We welcome potential errata on grammar and style if the changes improve readabil-
ity. Please note that First Aid style is somewhat unique; for example, we have fully
adopted the AMA Manual of Style recommendations on eponyms (“We recommend
that the possessive form be omitted in eponymous terms”) and on abbreviations (no
periods with eg, ie, etc).

The preferred way to submit new entries, clarifications, mnemonics, or potential correc-
tions with a valid, authoritative reference is via our website: www.firstaidteam.com.

This website will be continuously updated with validated errata, new high-yield con-
tent, and a new online platform to contribute suggestions, mnemonics, diagrams,
clinical images, and potential errata.

Alternatively, you can e-mail us at: firstaid@scholarrx.com.

NOTE TO CONTRIBUTORS
All entries become property of the authors and are subject to editing and reviewing.
Please verify all data and spellings carefully. Contributions should be supported by at
least two high-quality references.

Check our website first to avoid duplicate submissions. In the event that similar
or duplicate entries are received, only the first complete entry received with valid,
authoritative references will be credited. Please follow the style, punctuation, and
format of this edition as much as possible.
xxiii

00_USMLE-STEP1_00fm_i-xxiv.indd 23 9/17/18 2:27 PM


JOIN THE FIRST AID TEAM
The First Aid author team is pleased to offer part-time and full-time paid internships
in medical education and publishing to motivated medical students and physicians.
Internships range from a few months (eg, a summer) up to a full year. Participants will
have an opportunity to author, edit, and earn academic credit on a wide variety of
projects, including the popular First Aid series.

We are actively seeking passionate medical students and graduates with a specific
interest in improving our medical illustrations, expanding our database of medical
photographs, and developing the software that supports our crowdsourcing plat-
form. We welcome people with prior experience and talent in these areas. Relevant
skills include clinical imaging, digital photography, digital asset management, infor-
mation design, medical illustration, graphic design, and software development.

Please email us at firstaid@scholarrx.com with a CV and summary of your interest


or sample work.

xxiv

00_USMLE-STEP1_00fm_i-xxiv.indd 24 9/17/18 2:27 PM


1
Biochemistry

01_USMLE-STEP1_ch01_001-022.indd 1 9/17/18 3:09 PM


2 CHAPTER 1 Biochemistry

CASE 1
A 45-year-old man presents to his physician for chronic arthritis, which is worsening and affecting his lower back,
hips, and knees. On physical examination, you notice that the patient’s sclerae appear to have a brown discoloration;
however, his vision is unchanged from prior examinations. His ear cartilage is similarly discolored. An x-ray of the
spine reveals disk degeneration and dense calcification that is most prominent in the lumbar region. His urine
appears normal at first but begins to darken with time.

What is the most likely diagnosis?


Alkaptonuria (ochronosis).

What is the biochemical defect in this condition?


Alkaptonuria is characterized by the absence of homogentisate oxidase, an enzyme in the tyrosine degradation
pathway that catalyzes the conversion of homogentisate to maleylacetoacetate (see Figure 1-1). The accumulation
of homogentisate in cartilage leads to arthritis and causes discoloration of sclerae and other body tissues.
Phenylalanine
Phenylalanine
hydroxylase
Tyrosine
Tyrosine
aminotransferase
4-Hydroxyphenylpyruvic
acid
4-Hydroxyphenylpyruvic
acid dixoygenase
Homogentisate
Alkaptonuria
Homogentisate
1,2-dioxygenase
Maleylacetoacetic acid

FIGURE 1-1. Metabolic disorder in alkaptonuria. (Reproduced with permission from USMLE-Rx.com.)

The metabolite that accumulates in this condition is derived from which essential amino acid?
Homogentisate is a metabolite of phenylalanine. Homogentisate oxidase is necessary for phenylalanine degradation.
Homogentisate is further metabolized to acetoacetate and fumarate (part of the tricarboxylic acid cycle).

Given the extent of joint disease in this patient, how might his mental functioning be affected?
Alkaptonuria has no effect on cognitive functioning. Aside from its effects on joints and discoloration of the sclerae
and skin, the disease is benign.

What is the treatment for this condition?


There are no known ways to prevent the build-up of homogentisate. Dietary restriction of tyrosine and
phenylalanine reduces the production of homogentisate, but this approach has demonstrated no benefit on the
overall condition. Treating the symptoms of the patient’s arthritis is the only recommended therapy in this case.

Which amino acids are exclusively ketogenic, and which amino acids are glucogenic?
Leucine and lysine are the only purely ketogenic amino acids. Isoleucine, phenylalanine, threonine, tryptophan,
and tyrosine are both ketogenic and glucogenic. The remaining amino acids are purely glucogenic. Glucogenic
amino acids are those whose metabolites can be converted into glucose, through gluconeogenesis, whereas
ketogenic amino acids are degraded into acetyl-CoA, which can be converted to ketone bodies or acetoacetate.

01_USMLE-STEP1_ch01_001-022.indd 2 9/17/18 3:09 PM


Biochemistry CHAPTER 1 3
CASE 2
A 6-year-old girl presents to your office accompanied
by her mother, who is concerned about her daughter’s
recent difficulty walking. She states that her daughter
was born full term and there were no complications
during the pregnancy, but she has had frequent
respiratory infections throughout her life. When you
ask the girl to stand, you notice that she wobbles from
side to side. The eye exam reveals branching blood
vessels patterns (see Figure 1-2).

FIGURE 1-2. (Reproduced with permission from Chen Z, Ye W, Long Z,


et al. PLoS ONE. 2015;10(10):e0139738.)

What is the most likely diagnosis?


This girl is most likely suffering from ataxia telangiectasia (AT). AT is an autosomal recessive condition caused by
a defect in the ATM (AT mutated) gene. AT is characterized by a constellation of symptoms including difficulties
with balance or other cerebellar functions, recurrent respiratory infections due to IgA deficiency and a defect
in T-cell immunity. Lastly patients have a distinctive spider angioma termed telangiectasia, such as the ocular
telangiectasia seen in Figure 1-2.

What type of DNA damage is repaired less efficiently in patients with this condition?
Patients with AT are unable to efficiently repair double-stranded DNA breaks (DSBs) through a process called
nonhomologous end joining. Furthermore, checkpoint signaling and apoptosis in response to DSBs are defective.
This places individuals with AT at an increased risk for malignancies due to DSBs from sources such as ionizing
radiation.

Which phase of the cell cycle is affected by an ATM mutation?


The transition between the G1 and S phase of the cell cycle is most greatly affected by the ATM mutation. During
the S phase of the cell cycle, new DNA is synthesized from an intact parent strand template, so that daughter cells
will inherit identical copies of DNA. To ensure this, the cell scans the DNA for damage so that it can be repaired
prior to S phase DNA synthesis. The ATM protein is essential for activating the checkpoint that halts the cell cycle
(see Figure 1-3) from progressing into S phase by phosphorylating p53. p53, when phosphorylated, works with the
Rb protein to halt the progression of the cell cycle into S phase while these damaged areas of DNA can be repaired.
Without a proper ATM, this process is not halted, and mutations may be passed on to daughter cells. Improper
functioning of Rb or p53 is responsible for numerous cancers.
S checkpoint
Is environment favorable? Enter cell cycle,
proceed to S phase
h a s e
r p G
e 1,
t S
,G
n 2
S
I

G1
sis

G2
kine

M
it
o
Cyto

s
is

G2/M checkpoint
Is all DNA replicated?
Is environment favorable?
Enter mitotic phase

Metaphase to anaphase checkpoint


G0
Are all chromosomes attached to spindle?
Trigger anaphase, proceed to cytokinesis

FIGURE 1-3. Cell cycle checkpoints. (Reproduced with permission from USMLE-Rx.com.)

01_USMLE-STEP1_ch01_001-022.indd 3 9/17/18 3:09 PM


4 CHAPTER 1 Biochemistry

CASE 3
A 36-year-old firefighter races into a burning building trying to extinguish a fire. The building suddenly collapses,
and his fellow firefighters rush in to save him. They find him lying on the ground with his mask off. He is rescued
and transported to the nearest ED. You are the first medical student to see him (look how far you’ve come), and on
initial assessment you find that the man’s skin is bright red and he is breathing rapidly. You also notice that his breath
smells like bitter almonds.

What is the most likely diagnosis?


This man has been exposed to cyanide, which may be seen after exposure to burning buildings (the “bitter
almond” breath is pathognomonic for cyanide exposure). Carbon monoxide exposure may also present with red
skin but will not be associated with bitter almond smelling breath.

What biochemical process is disrupted in this condition?


Cyanide is an inhibitor of
cytochrome C oxidase (complex
IV) leading to disruption of the +
H+
H+

+H
electron transport chain (see + + +
+ + + + H+ +
+ + +
Figure 1-4). + + +
+ + +
+ + + + +
Innermembrane
+ + + Cyt c +
space +
+ + +

I e– e– IV
CoQ III Inner
mitochondrial
membrane
e– _
__ _ _ _ _ _
_ _
_ _ _ _ _ _
_ _ __ _ II
_ _ _
_ Mitochondrial
_
_ _ _ _ matrix _
_
NADH 2H+ + 1 O2 H2 O
2
V
FADH2 FAD

NAD+
ADP + Pi ATP + H2O
H+

FIGURE 1-4. Overview of electron transport chain and oxidative phosphorylation. (Repro-
duced with permission from USMLE-Rx.com.)

How does cyanide interfere with ATP production?


The electron transport chain generates a proton gradient across the inner mitochondrial membrane (see
Figure 1-4). ATP synthase (complex V) is driven by this proton gradient across the mitochondrial membrane. The
ingestion of cyanide inhibits complex IV, decreasing the ability of the electron transport chain to generate the
proton gradient. Without this gradient, ATP synthase cannot produce ATP.

What is the treatment for this condition?


Hydroxocobalamin (vitamin B12) and sodium nitrite are used to treat cyanide poisoning. Vitamin B12 binds
to cyanide, forming cyanocobalamin, which can then be excreted. Sodium nitrite oxidizes hemoglobin to
methemoglobin. This is normally undesirable because methemoglobin cannot bind oxygen less avidly. However,
methemoglobin strongly binds cyanide, causing it to release complex IV and preventing further disruption of the
electron transport chain.

What other substances inhibit the electron transport chain (ETC)?


Rotenone and amytal are inhibitors of complex I (NADH dehydrogenase). Antimycin A inhibits complex III
(ubiquinone cytochrome C oxidoreductase). Azide and carbon monoxide inhibit cytochrome C oxidase.
Oligomycin is an inhibitor of ATP synthase.

What substances act within the mitochondria and cause decoupling of the ETC?
Compounds such as 2,4 dinitrophenol and aspirin allow protons to leak across the inner mitochondrial membrane,
uncoupling ATP synthesis and oxygen consumption. The release of the proton gradient in a way that is not tied to
the ATP synthase enzyme leads to decreased ATP production.

01_USMLE-STEP1_ch01_001-022.indd 4 9/17/18 3:09 PM


Biochemistry CHAPTER 1 5
CASE 4
A 2-day-old boy is brought to the ED by his mother because of
frequent vomiting. The child was born to a 40-year-old mother who
reports that she did not undergo any prenatal testing, including
amniocentesis or karyotype analysis. The mother reports that since
birth, the boy has been vomiting greenish material immediately
after eating. He has also become lethargic and progressively less
responsive. On physical examination, the boy is found to have several
abnormalities, including prominent epicanthal folds, upslanting
palpebral fissures, and macroglossia. He also has thick skin at the
nape of his neck. X-ray of the abdomen is shown in Figure 1-5.

FIGURE 1-5. (Reproduced with permission from USM-


LE-Rx.com.)

What is the likely diagnosis?


The child most likely has Down syndrome, which can be associated with gastrointestinal disorders such as
duodenal atresia (as seen in Figure 1.5) or stenosis, annular pancreas, tracheoesophageal defects, and imperforate
anus. Duodenal atresia below the sphincter of Oddi causes bilious vomiting as seen in this patient.

What is the most common cytogenetic What screening is available in utero for this
abnormality in patients with this condition? condition?
Trisomy 21, resulting from nondisjunction of Markers of Down syndrome in maternal blood
chromosome 21 during meiotic anaphase 1 or include:
anaphase 2. The risk of nondisjunction increases • Reduced levels of α-fetoprotein, PAPP-A, and
with maternal age. estradiol
• Elevated levels of β-human chorionic
What other medical abnormalities are seen in gonadotropin (β-hCG) and inhibin
children with this condition? Ultrasound measurements displaying increased
A single palmar crease; small, folded ears; a short nuchal lucency may also been seen. A definitive
neck; Brushfield spots (pale yellow spots on the diagnosis can be made via karyotype analysis of
iris); and a gap between the first and second toes. fetal cells obtained via amniocentesis or chorionic
They also suffer from heart disease, most often villus sampling.
cardiac cushion malformations, and may have
ophthalmologic problems, gastrointestinal tract Later in life, what disorders is this baby at risk
malformations, poor hearing, cognitive disability, of developing?
and developmental delay. Males with Down Older individuals with trisomy 21 have a high
syndrome are almost always infertile. Individuals risk of developing early Alzheimer disease (on
with Down syndrome may also have atlantoaxial average by age 50). This may be because the
instability (increased movement between the C1 amyloid-β protein implicated in Alzheimer
and C2 vertebrae), which may lead to spinal cord disease is encoded on chromosome 21. They are
damage. Fetal alcohol syndrome, in comparison, also at increased risk of hematologic disorders,
is characterized by a thin vermillion border, small particularly acute leukemias, which occur in
palpebral fissures, and a smooth philtrum. childhood, most commonly acute lymphoblastic
leukemia.

01_USMLE-STEP1_ch01_001-022.indd 5 9/17/18 3:09 PM


6 CHAPTER 1 Biochemistry

CASE 5
A 7-year-old boy presents to your office accompanied by
his mother, who is concerned that her child has developed
difficulty walking. On your examination, you notice that the boy
walks with a wide-based gait and his calves appear larger than
expected (see Figure 1-6). When he gets up from your exam
table, he uses his arms to push himself up before standing.

FIGURE 1-6. (Reproduced with permission from


S­ enanayake HM, et al. syndrome: A case report. Int Arch
Med. 2014;7:2.)

What is the most likely diagnosis?


Duchenne muscular dystrophy (DMD). DMD is an X-linked recessive disorder caused by absence of the dystrophin
protein, leading to muscle weakness. A notable characteristic is pseudohypertrophy of the calves caused by
fatty replacement of the muscle fibers, as shown in Figure 1-6. Gowers maneuver is a physical exam finding
characteristic of DMD in which patients use their upper extremities to help them stand.

What type of mutation causes this patient’s condition?


Duchenne muscular dystrophy is caused by a frameshift mutation in the dystrophin gene. Frameshift mutations
are caused by the addition or deletion of a number of base pairs that is not divisible by 3, which shifts the reading
frame of the protein from the point of the mutation onward throughout the rest of the open coding region.
Remember that each amino acid is coded for by 3 bases, so any addition or deletion of base pairs that is not
divisible by 3 will affect all downstream amino acids. Patients may present with elevated creatine kinase levels,
likely secondary to increased muscle breakdown. Patients with DMD often develop dilated cardiomyopathy
leading to heart failure.

What other disease has a similar presentation but is typically milder?


Becker muscular dystrophy is caused by a mutation that leads to a partially functional dystrophin protein, as
compared to the nonfunctioning version seen in DMD. This leads to a less severe form of muscular dystrophy that
typically presents later in life. DMD presents as early as before 5 years of age.

01_USMLE-STEP1_ch01_001-022.indd 6 9/17/18 3:09 PM


Biochemistry CHAPTER 1 7
CASE 6
A 5-year-old boy is brought to his pediatrician because of frequent bruising, even after minor trauma. Other than
one episode of shoulder dislocation, his medical history is unremarkable. On physical examination the child has
many bruises at different stages of healing. He also has hyperextensible joints, flat feet, and dental crowding.

What is the most likely diagnosis?


Ehlers-Danlos syndrome. Although Marfan syndrome can cause joint hypermobility, it is unlikely to cause the
easy bruising seen in this patient. Most forms of Ehlers-Danlos syndrome are inherited as autosomal dominant
mutations. Zinc deficiency is another possible cause of poor wound healing since this micronutrient is needed for
proper collagen synthesis. However, zinc deficiency would be associated with other symptoms such as hair loss,
diarrhea, and possible acrodermatitis enteropathica, which is characterized by a perioral and acral rash.

What is the etiology of the easy bruising in this child?


Mutations that affect the formation of type III collagen may prevent proper synthesis or post-translational
modification of collagen. The bruising is a direct result of defects in the collagen of vessel walls.

What other abnormalities may be present in individuals with this condition?


Thin, fragile skin; abnormal scar formation; aortic aneurysms; rupture of large arteries; and rupture of the bowel
and uterus (pregnancy increases the risk of uterine rupture).

What are the stages of collagen synthesis?


As seen in Figure 1-7:
• Co-translational translocation of α collagen polypeptide by DNA
ribosomes docked on the rough endoplasmic reticulum. mRNA
• Hydroxylation of proline and lysine residues in the
endoplasmic reticulum. This step requires vitamin C.
• Glycosylation of lysine residues on the α chains in the Golgi Nucleus
apparatus, followed by assembly of three α chains to form a
triple helix of procollagen.
• The procollagen is secreted by exocytosis.
Hydroxylation
• The amino and carboxy pro-peptides are removed by
specific peptidases to form tropocollagen.
OH OH
• Many tropocollagen units line up in a staggered
arrangement and cross-link to form the mature collagen Glycosylation Endoplasmic
fibrils. (pro chain) reticulum

OH OH

Pla
sm
c(1–)
Exocytosis (peptide cleavage) a m
emb
rane
Collagen fibrils
with crosslinks

FIGURE 1-7. Collagen synthesis outside the cell. (Reproduced


with permission from USMLE-Rx.com.)

Where are the four major types of collagen found in the body?
• Type I: Bone, skin, tendons, fascia, dentin, and the cornea. Most common type of collagen.
• Type II: Cartilage, nucleus pulposus, and the vitreous body.
• Type III: Reticular collagen, found in skin, blood vessels, the uterus, granulation tissue, and fetal tissue.
• Type IV: Basement membranes.

01_USMLE-STEP1_ch01_001-022.indd 7 9/17/18 3:09 PM


8 CHAPTER 1 Biochemistry

CASE 7
A 27-year-old man with little prior medical care
was brought to the ED because of left-sided chest
pain followed by sudden collapse. Despite first
responders’ best efforts, resuscitation attempts
were unsuccessful. At autopsy the aorta is opened
lengthwise (see Figure 1-8). He was also noted to
have had small, raised yellow-brown lesions on
the extensor surfaces of his arms.

FIGURE 1-8. (Reproduced courtesy of Centers for Disease Control and


Prevention/Dr. Edwin P. Ewing, Jr.)

What is the most likely diagnosis?


This patient had familial hypercholesterolemia (FH), an inherited disorder characterized by extremely high serum
cholesterol levels that lead to the build-up of plaque, as seen in Figure 1-8. There are many different forms of
inherited hyperlipidemias (see Table 1-1).

Table 1-1. Inherited Hyperlipidemias


Hyperlipoproteinemia Defect Lipoprotein increased
I Lipoprotein lipase (LPL) Chylomicrons
IIa LDL receptor LDL
III Apo E2 synthesis IDL
IV VLDL elimination VLDL

Type I familial hyperlipidemia is caused by a lipoprotein lipase deficiency and results in abdominal pain,
xanthomas, and hepatosplenomegaly. Type III is caused by a defect in apolipoprotein E2 synthesis and results in
palmar xanthomas and tubo-eruptive xanthomas. Type IV is caused by increased very-low-density lipoprotein
(VLDL) production and decreased elimination. Low-density lipoproteins (LDL) transport cholesterol synthesized
in the liver to the tissues, whereas high-density lipoproteins (HDL) scavenge cholesterol from the tissues and
transport it back to the liver for disposal. Elevated LDL levels increase the risk of cardiac disease due to endothelial
damage when they induce an inflammatory cascade and accumulation of cholesterol-laden foam cells in walls of
blood vessels. In contrast, elevated HDL levels are mildly protective from such disease, because HDL can scavenge
cholesterol from the inflammatory plaques.

01_USMLE-STEP1_ch01_001-022.indd 8 9/17/18 3:09 PM


Biochemistry CHAPTER 1 9

What is the genetic pattern of this condition?


FH is inherited in an autosomal dominant manner. Heterozygotes typically have elevated cholesterol, approximately
370 mg/dL, and are at increased risk of myocardial infarctions. Homozygotes frequently have extremely high
cholesterol levels, up to 1000 mg/dL, and many die before 30 years of age from cardiovascular disease. Normal total
cholesterol is < 200 mg/dL, and levels > 240 mg/dL are considered elevated.

What is the molecular basis of this condition?


In FH there is a mutation in the LDL receptor gene that interferes with LDL uptake by the liver and other tissues.
Normally, LDL circulates in the blood and binds to its receptor on hepatocyte membranes and is then taken up
into the liver and metabolized. In FH patients, LDL is taken up by the hepatocytes less efficiently. Normally, cellular
cholesterol synthesis is down-regulated in response to high levels of serum cholesterol. Without LDL uptake, cells
have no way of sensing serum cholesterol levels, resulting in constitutive cholesterol synthesis and elevated LDL
levels in the blood.

What would the microscopic examination of the lesions on this patient’s arms show?
Cholesterol deposits in the skin, called xanthomas, form when there is a persistently elevated LDL level. They are
composed largely of lipid-laden macrophages.

Statin drugs are frequently used to treat this condition. What is their mechanism of action?
Statins (such as atorvastatin and rosuvastatin) inhibit 3-hydroxy-3-methylglutaryl coenzyme A reductase (HMG-
CoA reductase), a hepatic enzyme that catalyzes the rate-determining step in cholesterol synthesis. They reduce
the amount of endogenous cholesterol synthesized by the liver. This leads to the liver increasing the amount of
cholesterol it takes up from the bloodstream, which can then be used for processes such as steroid synthesis. This
decreases the amount of cholesterol in the bloodstream (see Figure 1-9).

Gut Liver Blood


Acetyl-CoA
LDL
HMG-CoA

Mevalonate LDL IDL

Cholesterol Lipoprotein
lipase
Bile acids
VLDL

FIGURE 1-9. Cholesterol chemical pathways in gut, liver, and blood. (Reproduced
with permission from USMLE-Rx.com.)

01_USMLE-STEP1_ch01_001-022.indd 9 9/17/18 3:09 PM


10 CHAPTER 1 Biochemistry

CASE 8
A 6-year-old boy is followed by his pediatrician because of delayed language acquisition and behavioral problems
at school. On physical exam, he appears to have a long face, large ears, and flat feet. His mother reports a normal
pregnancy with standard prenatal care and adds that she did not use drugs or alcohol during the pregnancy.

What is the most likely diagnosis?


This boy has fragile X syndrome. This disease occurs in individuals who have an expansion of a CGG trinucleotide
repeat sequence on the X chromosome. This expansion results in hypermethylation of DNA in the 5′ region of the
FMR1 gene, which silences the gene by inhibiting its transcription. The FMR1 protein is an RNA-binding protein.

What is the inheritance pattern of this condition?


Fragile X syndrome is an X-linked genetic disorder. It is the most common inherited cause of intellectual
disability. Down syndrome is the most common chromosomal cause of intellectual disability but is caused by
a chromosomal trisomy, most commonly caused by an error during meiosis, rather than a heritable genetic
sequence mutation. The hallmark of X-linked disorders is the absence of father-to-son disease transmission, since
fathers always pass a Y chromosome to their sons. These disorders are much more common in males than in
females. However, mild symptoms of fragile X syndrome appear in a significant minority of female carriers. Fragile
X syndrome is not fully penetrant, and many families show a maternal transmission pattern.

What physical abnormalities are associated with this condition?


Individuals with fragile X syndrome frequently have long, narrow faces with a large jaw, large ears, and a
prominent forehead. Most postpubertal males also have macro-orchidism.

What are other trinucleotide repeat disorders, and why are they associated with “premutation”?
Other disorders attributable to trinucleotide repeat expansion include:
• Huntington disease—CAG expansion on chromosome 4. Patients with Huntington disease present in their 30s
or 40s with dystonia, decreased cognition, aggression or changes in behavior, and chorea.
• Myotonic dystrophy—CTG expansion on chromosome 19. Myotonic dystrophy presents in adulthood and is
characterized classically with the muscle wasting and weakness due to the inability to relax muscles.
• Friedreich ataxia—FAA expansion on chromosome 9. Friedreich ataxia usually presents between the ages of
5 and 15 years with posterior cord and lateral corticospinal tract degenerations. Death is commonly due to an
enlarged heart.

Typically, higher numbers of trinucleotide repeats result in more severe and earlier onset of the phenotypic
expression of disease.
• Premutation occurs in patients with an intermediate number of repeats who are clinically normal but whose
children are at increased risk of expressing clinical disease.
• Anticipation is the worsening of disease through generations because of increasing number of trinucleotide
repeats.

01_USMLE-STEP1_ch01_001-022.indd 10 9/17/18 3:09 PM


Biochemistry CHAPTER 1 11
CASE 9
A 5-month-old girl is brought to the pediatrician by her parents because she has been very sleepy lately and has
been vomiting and sweating profusely at night. The infant’s mother says that their daughter was doing fine during
the first months of life but began showing these changes shortly after she began weaning from breast milk and
started drinking fruit juices. On physical exam, you palpate an enlarged liver. Laboratory testing reveals a serum
glucose level of 30 mg/dL. Urinalysis is negative for glucose but does note the presence of a reducing sugar.

What is the most likely diagnosis?


The most likely diagnosis is fructose intolerance given this patient’s presentation of symptoms after drinking fruit
juices, which have high levels of fructose.

What intermediate is elevated within the liver cells in this condition?


Fructose-1-phosphate is elevated in fructose intolerance.

What enzyme is deficient in this condition?


Aldolase B is deficient in this disorder.

How does this condition cause hypoglycemia?


Aldolase B catalyzes the conversion of fructose-1-phosphate into glyceraldehyde and dihydroxyacetone
phosphate (see Figure 1-10). Its absence results in accumulation of fructose-1-phosphate in liver cells and excess
serum fructose, which is excreted in the urine. Additionally, the accumulation of fructose-1-phosphate results
in depletion of inorganic phosphate and thus decreased ATP synthesis. Aldolase B also plays an essential role in
gluconeogenesis, so serum glucose levels are reduced in its absence.
Fructose

ATP
Fructokinase
ADP
Fructose-1-P
Aldolase B

Glyceraldehyde Dihydroxyacetone-P

NADH ATP
Triose
kinase
NAD ADP
Glyceraldehyde-3-P
Glycerol

Glycolysis

FIGURE 1-10. Fructose Intolerance and role of aldolase B. (Modi-


fied with permission from USMLE-Rx.com.)

What is the treatment for this condition?


The condition is treated through the removal of fructose, sucrose (a disaccharide of glucose and fructose), and
sorbitol from the diet.

Why did the infant exhibit no symptoms while exclusively fed breast milk?
Carbohydrates in breast milk derive largely from lactose comprised of glucose and galactose rather than fructose,
as compared to fruit juices, which have high levels of fructose.

01_USMLE-STEP1_ch01_001-022.indd 11 9/17/18 3:09 PM


12 CHAPTER 1 Biochemistry

CASE 10
A 12-year-old intellectually disabled boy is brought into a health clinic. His parents have noted that he seems to have
difficulty with his vision. Physical examination reveals bilateral dislocated lenses and a tall, thin body habitus with
especially long extremities. Laboratory studies show increased levels of serum methionine and serum homocysteine.

What is the most likely diagnosis?


Homocystinuria.

What is the biochemical defect in this condition?


The most common form of inherited homocystinuria results from reduced activity of cystathionine beta
synthase, an enzyme that converts homocysteine to cystathionine (see Figure 1-11).
Methionine Cystathionine
synthase synthase
Methionine Homocysteine Cystathionine Cysteine
B12 B6
Serine

FIGURE 1-11. Homocystinuria. (Reproduced with permission from Le T, et al. First


Aid for the USMLE Step 1: 2019. New York: McGraw-Hill, 2019.)

What vitamin supplementation is appropriate in this condition?


Vitamin B6 (pyridoxine) is a necessary cofactor with cystathionine synthase. Vitamin B6 supplementation has been
successful in many patients with this enzyme deficiency.

In addition to vitamin supplementation, what other dietary changes should be made?


The absence of cystathionine beta synthase means that cysteine cannot be formed from methionine. Therefore,
cysteine becomes an essential amino acid. This child should be given a diet low in methionine and high in cysteine.

This boy has a marfanoid body habitus and lens subluxation, two characteristics of this condition. For
which other conditions is this patient at greatly increased risk?
This child is at increased risk for cardiovascular disease. Elevated plasma homocysteine increases risk of coronary
artery disease, stroke, and peripheral artery disease because homocysteine is prothrombotic and increases the
risk of clot formation. He is also at risk for osteoporosis. Homocysteine inhibits collagen cross-linking and over
time can cause osteoporosis. Homocystinuria is distinguished from Marfan syndrome by the increased risk of
thrombosis, intellectual disability, and the downward displaced lens. Marfan syndrome does not present with
intellectual disability or increased risk of thrombosis, and the lenses in Marfan syndrome are displaced upwardly.

What enzyme deficiency is most likely to be found in a patient with increased serum homocysteine but
decreased serum methionine?
This could be caused by a deficiency of methionine synthase. This enzyme catalyzes the conversion of
homocysteine to methionine. Like patients with cystathionine synthase deficiency, these patients often have
central nervous system dysfunction and vascular disease.

01_USMLE-STEP1_ch01_001-022.indd 12 9/17/18 3:09 PM


Biochemistry CHAPTER 1 13
CASE 11
A 1-year-old boy is brought to the pediatrician because his parents have recently noted several abnormalities.
Although the child was developmentally normal at birth, he does not interact with others the way his older sister
did at the same age. His parents also notice that he has coarse facial features. Physical examination reveals skeletal
abnormalities and an umbilical hernia. Funduscopic examination shows corneal clouding. Additionally, his liver and
spleen are enlarged, and his joints feel stiff.

What is the most likely diagnosis?


Hurler syndrome.

What is the pathophysiology of this condition?


This syndrome results from a defect in α-L-iduronidase, an enzyme essential to the degradation of dermatan
sulfate and heparan sulfate. This disease is one of the mucopolysaccharidoses, a group of hereditary disorders
characterized by defects in glycosaminoglycan (GAG) metabolism. Features that distinguish this disorder from
the other lysosomal storage disorders include coarse facial features and corneal clouding. In Hurler syndrome, the
GAGs are not appropriately degraded in the lysosomes and are therefore deposited in various tissues. The disease
is inherited in an autosomal recessive manner.

What disease has a similar presentation but is typically milder?


Hunter syndrome is another mucopolysaccharidosis. It is due to a deficiency of iduronate sulfatase and has
X-linked inheritance. Unlike Hurler syndrome, Hunter syndrome does not present with corneal clouding, but
affected patients may exhibit aggressive behavior.

What are the typical findings on electron microscopy?


The lysosomal vesicles appear swollen. This is due to accumulation of partially degraded polysaccharides.

What key modification must be made in the Golgi apparatus for lysosomal enzymes, such as
𝛂-L-iduronidase, to be properly targeted to lysosomes?
Lysosomal enzymes must be covalently modified with mannose-6-phosphate (M6P) as they pass through the cis
Golgi network to be targeted to the lysosomes. These M6P groups are then recognized by M6P receptor proteins in
the trans Golgi network. Failure of these M6P groups to be added leads to I-cell disease.

01_USMLE-STEP1_ch01_001-022.indd 13 9/17/18 3:09 PM


14 CHAPTER 1 Biochemistry

CASE 12
A 35-year-old man visits a fertility specialist with his 27-year-old wife. They have been trying to conceive for more
than 13 months but have been unsuccessful. The husband has no previous children, but the wife has two children
from a prior marriage. Their past medical history is unremarkable except for repeated sinus infections and a chronic
cough in the husband. Physical examination reveals a point of maximum impulse located at the right fifth intercostal
margin.

What is the most likely diagnosis?


The fact that the wife has had prior children suggests that the cause of infertility lies with the husband. Given the
history, Kartagener syndrome (primary ciliary dyskinesia) is most likely. This is a genetic disorder with an autosomal
recessive inheritance pattern that affects flagella function in the sperm along with cilia in other areas.

What is the cause of their infertility?


Abnormality in dynein, which is an adenosine triphosphatase that acts as a molecular motor and is responsible for
retrograde transport of material along microtubules. In addition, it is required for movement of cilia and flagella. If
this enzyme is not functional, it results in immotile sperm.

What is the cause of the husband’s recurrent sinus infections?


The cilia of the respiratory epithelium require functional dynein for motility. Without it, they are unable to
transport bacteria and particles out of the respiratory tract. The retained particles and bacteria can lead to
infections as well as a chronic cough with sputum production.

What abnormality is most likely to be observed on x-ray of the chest?


Situs inversus may be seen on an x-ray of the chest displaying the heart predominantly on the right side of the
thorax. Situs inversus is a condition that presents with complete reversal of right-left symmetry. It may also show
bronchiectasis, with signs of dilated bronchioles.

What immunodeficiency is also caused by a mutation in microtubule polymerization?


Chédiak-Higashi syndrome is an autosomal recessive disorder caused by a defect in microtubule polymerization
resulting in impaired migration of immune cells, such as neutrophils, and impaired lysosome fusion, which results
in large granules visible within the cytoplasm (see Figure 1-12). These both contribute to immune deficiency.
Patients present with recurrent bacterial infections with staphylococci and streptococci. Neutrophils filled with
large granules may be seen on microscopy of patients with this condition.

FIGURE 1-12. Peripheral blood smear displaying large granules within cytoplasm of neutrophils in a patient with Chédiak-Higashi syn-
drome. (Reproduced with permission from Lozano ML, et al. Towards the targeted management of Chédiak-Higashi syndrome. Orphanet J Rare
Dis. 2014;9:132.)

There are several antimicrobial drugs that inhibit microtubule function. What are some examples?
Mebendazole and related drugs inhibit microtubule activity in helminths. Griseofulvin is an antifungal that acts
on microtubules. Several chemotherapeutic agents also interfere with microtubule function, such as vincristine,
vinblastine, and taxols such as paclitaxel.

01_USMLE-STEP1_ch01_001-022.indd 14 9/17/18 3:09 PM


Biochemistry CHAPTER 1 15

CASE 13
A 2-year-old boy is brought to the pediatrician by his mother, who is visibly upset. The mother reports that her son
has recently been biting his fingers and scratching his face incessantly. She says he was developmentally normal
for the first months of his life but has become increasingly irritable since about 3 months of age. The mother also
mentions that her son often has “orange-colored sand” in his diapers. Laboratory studies reveal a serum uric acid
level of 55 mg/dL. Urinalysis reveals crystalluria and microscopic hematuria.

What is the most likely diagnosis?


Lesch-Nyhan syndrome.

What is the biochemical defect in this condition?


Lesch-Nyhan syndrome is characterized by a deficiency in hypoxanthine-guanine phosphoribosyltransferase
(HGPRT).

What is the function of the deficient enzyme?


HGPRT plays a key role in the purine salvage pathway (see Figure 1-13), recycling hypoxanthine and guanine to
the purine nucleotide pool. In the absence of this enzyme, excess purine bases are degraded into uric acid, thus
causing hyperuricemia. Uric acid crystals in the urine give rise to the crystalluria and lead to the “orange-colored
sand” seen in the diaper.
Nucleic acids Nucleic acids

(deoxy) (deoxy) (deoxy)


Nucleotides
GMP IMP AMP
(deoxy) (deoxy) (deoxy)
Nucleosides Guanosine Inosine Adenosine
PRPP PRPP

Bases Guanine Hypoxanthine Adenine

Xanthine

Uric acid

FIGURE 1-13. Purine degradation and salvage pathway. (Reproduced with permission from USMLE-Rx.com.)

What is the treatment for this condition?


Allopurinol is a drug that inhibits xanthine oxidase, thus preventing the formation of uric acid from the more
soluble hypoxanthine and xanthine. Hypoxanthine and xanthine can more easily be excreted in the urine. Doses
should be titrated to normalize serum uric acid levels. To prevent self-injury, affected children often need lifelong
benzodiazepine or barbiturate sedation, restraints, and behavioral therapy. Allopurinol can also be used in the
prevention of gout flairs due to the same mechanism of action.

What associated conditions are likely to develop if this condition is not treated?
Kidney stones, renal failure, gouty arthritis, and subcutaneous tophus deposits will result if the disorder is left
untreated.

01_USMLE-STEP1_ch01_001-022.indd 15 9/17/18 3:09 PM


16 CHAPTER 1 Biochemistry

CASE 14
A 19-year-old college student comes to the university health clinic complaining of muscle aches. She recently
began an exercise program to lose the 6-8 kg (13-17 lb) that she gained over the past year. After her first day of
weightlifting, however, she became extremely sore. Several hours later, her urine was the color of “cherry soda pop.”
Physical examination is unremarkable. Laboratory tests reveal a serum creatine kinase level of 93970 IU/L. Urinalysis
is negative for blood and positive for myoglobin.

What is the most likely diagnosis?


McArdle disease (type V glycogen storage disease). There are several glycogen storage diseases (see Table 1-2).

Table 1-2. Glycogen Storage Diseases


Glycogen storage disease Defect Characteristics
I-Von Gierke disease Glucose-6-phosphatase Fasting hypoglycemia
II-Pompe disease Alpha 1,4 glucosidase Cardiomegaly
III-Cori disease Glycogen debranching enzyme Hepatomegaly and normal lactate
IV-Andersen disease Glycogen branching enzyme Hepatosplenomegaly
V-McArdle disease Muscle glycogen phosphorylase Muscle pain and cramps

Other glycogen storage diseases include Von Gierke disease, which causes severe fasting hypoglycemia; Pompe
disease, which is characterized by cardiomegaly and early death; and Cori disease, which is less severe than Von
Gierke and has normal lactate levels.

What is the biochemical defect in this condition?


McArdle disease is caused by a deficiency of muscle glycogen phosphorylase. Although glycogen formation is not
affected, glycogen cannot be broken back down to glucose-1-phosphate via glycogenolysis.

What are the most likely liver and muscle biopsy findings?
A liver biopsy is normal, as the defective enzyme is present only in muscle. Muscle biopsy shows accumulation of
glycogen.

After the patient completes an exercise tolerance test, her lactic acid levels do not increase normally. Why?
Lactic acid is a product of anaerobic glucose metabolism, and during intense exercise, glucose in the muscle
comes from muscle glycogen stores. Failure of lactic acid levels to elevate after exercise is an indication of a defect
in the metabolism of glycogen or glucose to lactate. This response can be seen in other disorders of glycogenolysis
or glycolysis as well.

What accounts for the color of her urine?


During exercise her muscles begin to break down, a process known as rhabdomyolysis, because of the lack of
glucose. This causes myoglobinuria as well as elevated creatine kinase. Her urine will be positive for blood due to
cross-reactivity with myoglobin and hemoglobin on urine dipstick but will be negative for RBCs.

What is the treatment for this condition?


Oral ingestion of sucrose before exercise has been demonstrated to improve exercise tolerance and reduce the risk
of myoglobinuria. The patient should also warm up gently prior to exercise and avoid intense, anaerobic exercise.

01_USMLE-STEP1_ch01_001-022.indd 16 9/17/18 3:09 PM


Biochemistry CHAPTER 1 17

CASE 15
A 2-year-old boy is brought to a health clinic in Mexico because of poor development as well as vomiting, irritability,
and a skin rash. The boy’s mother also notes that his urine has a strange “mousy” odor. Physical examination reveals
the child has an eczema-like rash, is hyperreflexive, and has increased muscle tone. He has a surprisingly fair-skinned
complexion compared to the rest of his family. Laboratory studies reveal a serum phenylalanine level of 28 mg/dL.

What is the most likely diagnosis?


Phenylketonuria (PKU).

What is the pathophysiology of this condition?


PKU is caused by a defect in the metabolism of phenylalanine (see Figure 1-14). This essential amino acid is
converted to tyrosine by phenylalanine hydroxylase. However, when phenylalanine hydroxylase activity is reduced
or absent, phenylalanine builds up. This leads to excess phenyl ketones in the blood, resulting in the symptoms
seen in this patient. PKU is inherited in an autosomal recessive fashion.

COO– Phenylalanine hydroxylase COO–

H 3N + CH + O2 H 3N + CH

CH2 H4 Biopterin H2 Biopterin CH2

Dihydrobiopterin
reductase
NADP+ NADPH + H+ OH
Phenylalanine Tyrosine
FIGURE 1-14. Phenylalanine hydroxylase reaction. (Modified with permission from USMLE-Rx.com.)

What additional physical characteristics are common at presentation?


Affected children are normal at birth but fail to reach developmental milestones. Other physical findings include
failure to thrive and cognitive disability.

What is the cofactor for the defective enzyme in this disease that, when deficient, can also lead to increased
levels of phenylalanine in the blood?
A deficiency in tetrahydrobiopterin can also lead to increased blood levels of phenylalanine. Tetrahydrobiopterin is
a required cofactor for the synthesis of several neurotransmitters, including norepinephrine, dopamine, serotonin,
and the molecule nitric oxide. Deficiency leads to a buildup of phenylalanine, which can be toxic to the brain,
as well as an inability to produce those neurotransmitters. Tetrahydrobiopterin deficiency presents with similar
symptoms to phenylketonuria, along with progressive neurologic disabilities. It is inherited in an autosomal
recessive pattern.

What is the treatment for this condition?


PKU is treated with decreased dietary phenylalanine (which is contained in many foods, including artificial
sweeteners). In patients with PKU, tyrosine cannot be derived from phenylalanine, so it becomes an essential
amino acid. Therefore, patient should also receive dietary tyrosine supplementation. Currently, screening is
mandatory and performed 6 days to 2 weeks after birth, using high-performance liquid chromatography.

01_USMLE-STEP1_ch01_001-022.indd 17 9/17/18 3:09 PM


18 CHAPTER 1 Biochemistry

CASE 16
A 6-month-old girl is brought to the pediatrician because she has been feeding poorly and has been lethargic for
the past several months. The baby has also started breathing more rapidly than normal and recently had a seizure.
Laboratory studies reveal a serum pH of 7.20, an anion gap of 19 mEq/L, elevated levels of pyruvate and alanine, and
decreased levels of citrate.

What is the most likely diagnosis?


Pyruvate dehydrogenase deficiency.

What is the pathophysiology of this condition?


Glycolysis is the pathway that converts one molecule of glucose into two molecules of pyruvate. Pyruvate
dehydrogenase then converts pyruvate to acetyl-CoA, which can enter the tricarboxylic acid (TCA) cycle (see
Figure 1-15). Without this enzyme, cells are unable to fully oxidize glucose and must derive adenosine triphosphate
(ATP) from glycolysis alone. Pyruvate must be converted to lactate to regenerate oxidized nicotinamide adenine
dinucleotide (NAD+), a necessary cofactor in glycolysis. The elevated lactate level is responsible for the acidemia
and anion gap observed in this baby.
Glucose

αKG Glu NADH NAD+

Alanine Pyruvate Lactate


Lactate
Alanine
transaminase dehydrogenase

CO2 + ATP Pyruvate


dehydrogenase
NAD+
ADP + Pi
Pyruvate NADH
carboxylase
CO2
Oxaloacetate Acetyl-CoA

FIGURE 1-15. Pyruvate metabolism pathways. (Modified with permission from USMLE-Rx.com.)

Why are alanine levels high and citrate levels low in this condition?
Alanine levels are high because much of the excess pyruvate in the muscle is converted to alanine in a reversible
reaction by alanine aminotransferase. Citrate levels are low because there is little acetyl-CoA to combine with
oxaloacetate to form citrate.

What is the treatment for this condition?


Treatment involves increased intake of ketogenic nutrients (foods with high fat content). The breakdown of
fatty acids by β-oxidation involves reduction of flavin adenine dinucleotide (FAD) and NAD+ and produces one
molecule of acetyl-CoA for every two carbon atoms in the fatty acid chain. The FADH2 (1-5-dihydro-FAD) and
NADH (reduced NAD+) can be used by the electron transport chain to produce ATP, whereas the acetyl-CoA can
enter the TCA cycle. Oral citrate is also helpful for replenishing the substrates of the citric acid cycle.

Which are the only purely ketogenic amino acids?


Leucine and lysine are the only purely ketogenic amino acids.

Why is an anion gap metabolic acidosis seen in this condition?


Anion gap metabolic acidosis is caused by the accumulation of unmeasured anions (such as lactic acid). The anion
gap is defined as the difference between sodium in the serum and the sum of bicarbonate plus chloride. The body
buffers this excess acid with bicarbonate, decreasing the amount of measured bicarbonate on labs. Because lactic
acid is not a directly measured anion, this increases the anion gap leading to an anion gap metabolic acidosis.

01_USMLE-STEP1_ch01_001-022.indd 18 9/17/18 3:10 PM


Biochemistry CHAPTER 1 19

CASE 17
A 5-month-old girl of Ashkenazi Jewish descent is brought to her pediatrician because of concerns about
developmental regression. Although she was developing normally for the first 4 months, she can no longer roll over
by herself. In addition, she often smiled at 3 months of age but no longer does so. Funduscopic examination reveals
a cherry-red spot in the macula. The remainder of her physical examination is normal.

What is the most likely diagnosis?


Tay-Sachs disease.

What is the biochemical defect in this condition?


This disease, one of the sphingolipidoses, is caused by a deficiency of hexosaminidase A. This enzyme is present
within the lysosomes of central nervous system cells and helps degrade a lipid called GM2 ganglioside. GM2
ganglioside accumulation within the neurons leads to progressive neurodegeneration (see Figure 1-16). Children
become blind and deaf before paralysis ultimately sets in. Children with Tay-Sachs disease usually die by 3 years
of age.
Ceramide
GM2 Hexosaminidase A
trihexoside
Tay-Sachs Fabry α-galactosidase A
Glucocerebrosidase
(β-glucosidase)
GM3
Arylsulfatase A
Sulfatides Galactocerebrosidase
Metachromatic Glucocerebroside Sphingomyelinase
leukodystrophy Gaucher

Galactocerebroside Krabbe Ceramide Niemann-Pick Sphingomyelin

FIGURE 1-16. Biochemistry pathway of sphingolipidoses; lysosomal storage


diseases. (Reproduced with permission from USMLE-Rx.com.)

How is the gene responsible for this condition inherited?


Tay-Sachs disease is inherited in an autosomal recessive fashion. Fabry disease is the only one of the
sphingolipidoses that is inherited differently; it is X-linked.

What other conditions present with similar physical examination findings?


Niemann-Pick disease, which is caused by a deficiency of sphingomyelinase, also presents with a cherry-red spot
in the macula in approximately 50% of cases. These patients often present with anemia, fever, and neurologic
deterioration. The prognosis of Niemann-Pick disease is poor as well; most patients die by 3 years of age. Unlike
Niemann-Pick, Tay-Sachs does not involve hepatosplenomegaly and demonstrates onion-like lysosomes on
microscopy. Foam cells are characteristic of Niemann-Pick.

Which of the other sphingolipidoses also has a higher prevalence among Ashkenazi Jews?
Although Tay-Sachs is considered to have a higher prevalence among Ashkenazi Jews, screening programs have
significantly decreased the prevalence of Tay-Sachs in this group. Gaucher disease, which is caused by a deficiency
of β-glucocerebrosidase, also has a much higher incidence in this population.

01_USMLE-STEP1_ch01_001-022.indd 19 9/17/18 3:10 PM


20 CHAPTER 1 Biochemistry

CASE 18
A 36-year-old homeless man presents to a community health clinic complaining of increasing shortness of breath.
On questioning, the man admits to an extensive history of alcoholism. A review of systems reveals he has also
experienced tingling and burning in his legs for the past several weeks. Physical examination reveals that he is
tachycardic (heart rate of 122/min), has rales bilaterally, and has bilateral pitting edema. He also has decreased
sensation in his feet and is hyporeflexive in his lower extremities. An x-ray of the chest shows an enlarged cardiac
silhouette and bilateral pulmonary congestion.

What is the most likely diagnosis?


Vitamin B1 (thiamine) deficiency. Although the patient’s alcoholism presents a clear etiology, arsenic poisoning
also blocks thiamine utilization and can result in a clinical picture resembling thiamine deficiency and should also
be considered.

What clinical manifestations are commonly present in this condition?


This patient has the symptoms of both wet and dry beriberi. Patients with wet beriberi present with high-output
congestive heart failure and dilated cardiomyopathy. Patients with dry beriberi present with peripheral neuropathy
consisting of muscular atrophy and diminished sensation and reflexes. Dry beriberi presents similarly to vitamin B12
deficiency; however, vitamin B12 deficiency is usually due to a malabsorptive process, does not cause congestive
heart failure, and will cause a macrocytic anemia.

The deficient factor in this condition is a cofactor for which enzymes?


Thiamine is part of thiamine pyrophosphate (TPP). TPP acts as a cofactor for transketolase (an enzyme in the
hexose monophosphate shunt), pyruvate decarboxylase (a component of the pyruvate dehydrogenase complex)
(see Figure 1-17), and α-ketoglutarate decarboxylase (a component of the α-ketoglutarate dehydrogenase
complex).

TPP
Lipoic acid
FAD

CoA NAD

FIGURE 1-17. Conversion of pyruvate to acetyl CoA with necessary cofactors.


(Modified with permission from USMLE-Rx.com.)

What other pathologies are commonly seen with this condition?


Wernicke encephalopathy is the central nervous system manifestation of thiamine deficiency. This disease
classically consists of nystagmus, ophthalmoplegia, and cerebellar ataxia. When the additional symptoms of
confusion/psychosis and confabulation are seen, the disease is known as Wernicke-Korsakoff syndrome. It is
standard practice to give thiamine before glucose to any patient with suspected thiamine deficiency to prevent
Wernicke-Korsakoff. Administration of glucose prior to thiamine may worsen symptoms of thiamine deficiency due
to an inability to complete enzymatic reactions that rely on thiamine and conversion of pyruvate to form lactic acid
instead of entering the TCA cycle as acetyl-CoA.

What are the most likely MRI findings?


Although degenerative changes are often seen in the cerebellum, brain stem, and diencephalon, atrophy of the
mammillary bodies is most commonly noted.

01_USMLE-STEP1_ch01_001-022.indd 20 9/17/18 3:10 PM


Another random document with
no related content on Scribd:
helpless and friendless from the prisons of infidels, of the widows
and orphans of Moslems, of the poor and the sons of the highways,”
under certain conditions. It concludes with the declaration that
whoever infringes these conditions in the future “will do himself
injustice, will prove himself an infidel, and partaker in the blood of
Hassan and Husein, and an accomplice of those who do despite to
the statutes of God.” Then comes the signature—“the humble
servant of God, Serjenk.”
The church and dwelling-house of Boheira are shown here. Of
Boheira it is said that he was a monk in this city, and was the first to
hail the youthful Mohammed as a coming prophet. Of this event,
Ockly has translated the following account given by one Basil: “The
caravan of the Koreish came by, with which were Kadijah’s camels,
which were looked after by Mohammed. He [Boheira] looked towards
the caravan, in the middle of which was Mohammed; and there was
a cloud upon him to keep him from the sun. Then the caravan
alighted, and Mohammed leaning against an old withered tree, it
immediately brought forth leaves. Boheira perceiving this, made an
entertainment for the caravan, and invited them into the monastery,
Mohammed staying behind with the camels. Boheira, missing him,
asked if there were all of them. Yes, they said, all but a little boy they
had left to look after their things and feed the camels. ‘What is his
name?’ says Boheira. They told him Mohammed ibn ʿAbdullah.
Boheira asked if his father and mother were not dead, and if he was
not brought up by his grandfather and uncle. Being satisfied that it
was so, he said: ‘O Koreish! set a great value upon him, for he is
your lord, and by him will your power be great both in this world and
in that to come: for he is your ornament and glory.’ They asked him
how he knew that. ‘Because,’ answered Boheira, ‘as you were
coming, there was never a tree nor a stone nor a clod but bowed
itself and worshipped God.’ Boheira, besides, told this Basil that a
great many prophets had leaned against this tree, and sat under it,
but it never bore any leaves before since it was withered. ‘And I
heard him say,’ says this same Basil, ‘this is the prophet concerning
whom ʿIsa [Jesus] spake, Happy is he that believes him, and follows
him, and gives credit to his mission.’”
That Mohammed met Boheira seems certain. But exactly what their
relations were it is not easy to say. The Syrian Christians believed
that he followed “the prophet,” and largely assisted him in the
composition of his “messages” or “revelations.” They say he supplied
the biblical information used for Mohammed’s purposes in the
Korʾân. The number of Jews, however, who long ere Mohammed’s
time had settled in el-Yemen suggests a more convenient source for
his knowledge, such as it was, of the Torah; but for his acquaintance
with Christianity he may possibly have been indebted to some
renegade like Boheira. And if his (Boheira’s) understanding of the
words of Jesus be illustrated in the phrase quoted above, what
wonder if his religion did not greatly impress such a mind as
Mohammed’s! In any case, it was in his Syrian journeys that he must
have come into contact with Christianity. However bootless, it is
impossible to help regretting that the master mind of the Arabian
peninsula should have seen our religion only in the debased form
then prevalent in these regions. Had it been ordered otherwise, the
whole history of the East might have run in nobler channels.
Close by a second Roman archway stands a large ruined house,
abounding in carved and sculptured stones, known as Kasr-Melek
el-Asfar. At Zorʿa there is also a “palace of the yellow king.”
The fortress is built around and upon the old Roman theatre, which,
contrary to expectation in such circumstances, is well preserved.
There are vast underground apartments, and cisterns which would
supply water for a large garrison through a siege of many months.
Subterranean passages, the natives say, lead to a great distance in
several directions.
Such an important place as Bozrah was bound to claim to be the
birthplace of Philip; nor would it be complete without some relation to
Job. Accordingly, an “ancient tradition” is forthcoming to the effect
that the patriarch dwelt in the country near the city.
On Saturday afternoon we heard the sounds of music and drums
proceeding from the town, and high over all the peculiarly shrill,
wavering cry uttered by Eastern women in times of excitement,
whether of grief or joy. The tramp of horses on the pavement, and
the tread of many feet, told of the approach of a procession. Soon a
company of horsemen swept into view, youthful, well mounted,
armed with the long rumh, or Arab spear, accompanied by a crowd
of all ages, clad in holiday attire of brilliant colours. Riding in the
procession were several little boys, who seemed to have little
interest in the affair, and to be, on the whole, not a little bored by it. It
was a bridal procession—an occasion of special joy, since not one
but four marriages were being celebrated. The enthusiasm lacking
on the part of the dressed-up, solemn-looking little bridegrooms was
made up for amply by the excited people who surrounded their
horses, dancing, singing, shouting, and clapping their hands. A band
with drums and timbrels went in front. On the wrists and ankles of
the women glittered rude bracelets; heavy rings ornamented their
fingers; nor was the nose-jewel entirely absent. Their heads were
covered with light kerchiefs of varied hues, the corners tied under the
chin, while the hair hung down in long, heavy plaits behind, often
loaded with coins, which might be the dowry of the women who wore
them. The men wore the kufîyeh and ʿakal—the “kerchief” and “thick
hair fillet”—on their heads, with the Arab coat of goats’-hair over their
under-garments. Those who were not barefooted wore the common
red shoes so dear to the Arab.
The horses pranced and capered. The procession advanced with
singing, clapping of hands to the music, and at times in a kind of
stately dance. They headed toward a wide stretch of level ground
behind our camp. Passing within the enclosure, those on foot drew
themselves up along one side; the horsemen dashed forward at full
gallop, and began a series of evolutions which, to Western eyes,
seemed to involve every man of them in imminent danger. Not a few
of the performances in which they pride themselves are obviously
cruel to the animals. Riding at full speed, it is a mark of
horsemanship to bring the animal to an absolute stop in an instant,
throwing him back on his haunches. In starting, he must spring
forward at full speed, like an arrow from the bow. If either of these
movements cannot be performed, horse or rider, or both, are
condemned. In driving the horses peculiar spurs are employed. The
bottom of the Arab stirrup is a broad piece of light iron, the hinder
part of which is sharpened. When the foot is slipped forward, this
piece of iron projects behind the heel. Driven into the sides of the
animal, it cuts almost like a knife. And another mark of horsemanship
is that these cuts be as far back as possible. The bridle, too, is an
instrument of torture. From the centre of the bit a sharp piece of iron
projects inward; a ring attached to the same point drops over the
under jaw; the reins are attached to iron rods, which, from the ends
of the bit, extend a little way in front of the horse’s mouth, forming
thus a curb of terrific power. It is with this instrument the rider can
arrest his horse in a moment in mid-career. One can hardly help
wishing that, for the sake of the poor animal, he had a touch of its
quality himself. An exceptionally “hard-mouthed” horse may require
exceptional treatment, but the universal employment of this bridle
seems gratuitous cruelty.
Many of their feats, however, are very graceful, and in their
performance no little skill is required. Their beautiful wheeling and
curving on the level, in which horse and rider seem moulded
together, remind one of nothing so much as the fine circlings of an
expert skater. Good proficiency is attained when the rider can stoop
from the saddle at full gallop and pick up his staff from the ground. In
this and similar exercises the horseman on the medân, or
racecourse, is always engaged, in intervals of play. Loud were the
challenges of the men of Bozrah that festal day, and hearty the
responses. Prancing forward, one would touch another with his
rumh, and, turning, spur his steed and fly, hotly pursued by the man
thus challenged. Then ensued a series of evolutions in which all the
skill of the horsemen and all the speed of the horses were brought
into play. If the pursuer could put his rumh on the shoulder of his
challenger, he received the victor’s meed of applause; but should the
challenger’s steed outstrip that of the pursuer, the latter swerved off,
and sought to redeem his defeat by a display of skilful
horsemanship; and he might count himself fortunate if he reached
his place again without a second touch from his conqueror’s rumh.
The play over, the procession formed again, the solemn-looking little
men in the centre, as before, marched back to the city with music
and dancing, and passed away from our sight. We heard, at
intervals, the distant roll of the drums and the shrill cry of the women,
from which we knew that the festivities were still going on.
On Sundays more than other days one was impressed with the
abnormal quiet reigning over the land. Verily, the word of the Lord
has been fulfilled: “And you will I scatter among the nations.... Then
shall the land enjoy her sabbaths, as long as it lieth desolate, and ye
be in your enemies’ land, ... even the rest which it had not in your
sabbaths when ye dwelt upon it.” Temple, church, and mosque have
risen in succession in her cities, have flourished awhile in splendour,
then crumbled into ruin. At last the sabbath rest has fallen upon her.
The name Bozrah signifies a fortress, and must have described the
city from very ancient times. It would be of the highest importance to
dwellers in the cultivated lands to have here, on their border, a
strong defence against the wild rovers from the deserts. So true is
this, that it came to be said, “The prosperity of Bozrah is the
prosperity of Haurân.”

BOZRAH
This city is probably intended by Jeremiah as included in the
denunciation of wrath against Moab and all her cities, “far or near.” It
is mentioned in close connection with Kerioth and Beth Gamul,
which correspond as nearly as may be in name to Kerîyeh and Umm
el-Jamâl—ruined cities in the neighbourhood. Bozrah appears in the
Apocrypha and Josephus as Basora. Hither came the heroic Judas
Maccabeus. He delivered from imprisonment many of his
unfortunate brethren, and destroyed the city, burning it, as far as that
was possible, with fire. But withal there were yet glorious days in
store for Bozrah. The land passed under the dominion of the
Romans. The transjordanic provinces were subdued by Aulus
Cornelius Palma. Bozrah he made capital of the province, calling it
Nova Trajana Bostra, in honour of the emperor Trajan. This was in
a.d. 105, from which date was reckoned the Bostrian era. The old
city took a new lease of life, and worthily assumed her place as by
far the most important stronghold east of Jordan. Her streets were
graced with public buildings of which the proudest city need not have
been ashamed. A network of magnificent roads, which even yet are
traced across the plains, leading to all the principal towns and cities
in the province, found in her its centre. The merchandise of the East,
by way of the road from the Persian Gulf, stocked her marts, and the
gold and frankincense caravans from Arabia the Happy brought their
stores to increase her wealth. The time of her greatest splendour
probably fell in the short reign of Philip the Arabian, who, with the
wealth of Rome at his command, guided by his Oriental pride and
taste, would lavish adornment on the chief city of his native province,
embellishing her streets and squares with triumphs of architectural
art. Hither came the great Origen, to consult with the Bishop
Beryllus, who had gone astray in matters of faith and doctrine; and
he met with far greater success than the most doughty warriors for
orthodoxy may ever hope for, if they regard the heretic as one only to
be hunted out and prosecuted. Something might be learned from the
methods of Origen, who, in brotherly and friendly converse,
convinced the erring bishop, and saved him to the Church.
Subsequently the city became the seat of an archbishop. It
maintained its fame as a commercial and military centre down to the
Mohammedan conquest. Bozrah was the first Syrian city invested by
the Arabians. The intrepid and skilful soldier, Khâlid, surnamed “The
sword of God,” commanded the Moslems. While planted before the
fortress, the Mohammedans, in the absence of water, performed
their ablutions with sand. In the first encounter with Christians
outside the walls, the Moslems were entirely victorious. The former
shut themselves up in the city, and mounted banners and crosses
upon the walls, as if expecting divine intervention, by this means, in
their favour. The governor, Romanus, counselling surrender, was
deposed as a traitor, and another put in his place. Smarting under
the double insult, he resolved to revenge himself by selling the city to
the enemy. Through his treachery many valorous Moslem youths
were introduced into the city, in the garb of ordinary citizens, and
posted in various quarters. At a given signal, the Moslem war-cry,
Allah Akbar, resounded over the city. The defenders were thrown
into confusion, the inhabitants into consternation and despair. The
gates were thrown open, and the city, with little bloodshed, passed
into the hands of the Mohammedans, who retain it still. In the days of
the Crusades it was still an important stronghold, practically the key
to the possession of the eastern provinces. Baldwin III. cast his
forces in vain against the rock-like walls. But the blight of Islâm had
fallen upon it. Gradually its splendour faded; its well-stocked marts
were emptied; the sound of busy footsteps on its pavements died
away; earthquakes shook down its temples and destroyed its public
buildings; no hand was raised to arrest its decay. And now for
centuries it has lain mouldering in mournful ruins under the fierce
heat of the Syrian sun, blackening in the breath of Time. But surely
her season of solitude and desolation must be nearly over. When the
long sabbath of the land is past, new life pulsing in all her furrows,
the hills and vales resounding with the song of the husbandman,
Bozrah must awake from her weary sleep, and put on once again the
pleasing garments of prosperity.
CHAPTER IX
Travellers’ troubles—A corner of the desert—The mirage—
Dangerous wadies—Lunch in the desert—A “blind” guide
—The clerk to the sheyûkh—A milestone—Kalʿat Esdein
—Thirst—The uplands of Gilead—Search for water—A
Bedawy camp—Terrific thunderstorm.
Long before dawn on Monday morning all was bustle and stir in the
camp. We hoped to reach Jerash that evening, but the most
conflicting accounts were given of the distance, varying from three
days to one long day. The usual road runs west to Derʿat, where it
turns southward by way of Remtah. A line direct, across a corner of
the desert, is shorter by perhaps fifteen miles. This we proposed to
take. In that wide empty land, with never a house, haunted by roving
Beduw, a guide was absolutely necessary. With difficulty one was
found who had traversed the way before; but he would go only on
condition that a friend should also go, to accompany him home
again. We were not yet to start, however. A vendor of antiquities
entrusted certain old coins, seals, etc., to our cook, who himself did
business in that line, in the hope that we might buy. A few purchases
were made; but when it came to giving back the remainder, a seal, or
stone from a signet ring, was missing. On this, of course, the owner
put a fancy price. Imagine a company of pilgrims on their knees,
turning up stones and groping in the dust as earnestly as rag-pickers
on a heap! The toil was fruitless. The cook was told that suspicion
attached to himself; and that if the seal were not forthcoming, the
owner should have his price, the same to be duly deducted from the
cook’s wages. With an injured air that plainly meant “What shall we
hear next?” the worthy ʿAbdu resumed his search, and soon sprang
to his feet with the lost seal in his hand. Throwing himself down, he
kissed the ground, then casting his eyes upward he fervently
exclaimed, el-hamdulillah, “Praise be to God!” The owner seemed
least pleased of all. Tying up his treasure in the corner of a napkin,
he marched sullenly away, grieving doubtless over ʿAbdu’s provoking
luck.
At last our guide strode off before us, leaving his companion to fetch
the muleteers, who, we hoped, might pass us at lunch. We struck the
Roman road which runs to the south-west, not that which leads more
to the south, past Umm el-Jamâl to Kalʿat ez-Zerka. The pavement
on these great highways is hard on the horses’ hoofs. The track
used to-day almost invariably lies alongside the road. Crossing a
shallow vale, we entered a vast plain, covered with tufts of wiry
grass. The beautiful iris was here also in plenty. The view offered
little variety save on the horizons. Salchad, Jebel el-Kuleib, and the
dark range of which they are part loomed away on the north-east.
Northward lay el-Lejâʾ and the mountains that overlook Damascus.
The white splendour of Hermon filled the north-western sky. A light
haze half-concealed the hills of Jaulân and the highlands of Galilee,
but nearly due west we could see the round head of Tabor. Before us
lay the wooded hills of Jebel ʿAjlûn, the land of Gilead, while the
plain stretched away to the south-east, desert-wards, as far as the
eye could reach.
Several times during this desert ride we saw the mirage—now as
waving trees, now as dimly outlined houses, with the sheen of water
near. Happily we had supplies of water with us, and so were spared
the torture these fleeting visions bring to the weary and the thirsty.
The mirage is often seen in the plain of el-Bukaʿ, or Coele Syria.
One most perfect and beautiful I saw in the neighbourhood of Tell
Hûm, from a roof in Tiberias. It proved to be a picture of Tiberias
itself, with ruined castle, broken walls, white-domed mosque, and
palm trees, photographed upon the mists some nine miles away.
We crossed several little wadies in which the water from winter and
spring rains had not quite dried up. The passage of these “brooks” is
not always free from danger. The soft soil goes to thick black mud,
when saturated with the water of the stream. Several of our party
had narrow escapes from accident, the horses sinking to the saddle-
girths, and struggling through only with desperate efforts, very
unsettling to the riders. In the deepest and broadest of these hollows
we found a green-carpeted meadow, with a few Bedawy tents. The
moment we came in sight a woman ran out to meet us, with
hospitable welcome, bringing a large dish of buttermilk, from which
we drank heartily and were refreshed.
At this point I was some distance behind the party, detained in
digging a root from the hard soil. On the north of the wady lay a
rough hill, strewn with great boulders. Galloping up, I saw two figures
with long guns dodging behind the rocks, furtively glancing in my
direction, and clearly making to intercept me in the wady. A few
steps farther, and they caught sight of our party, who fortunately
were not very far away. They at once turned and made off to
eastward, so sparing me what might have been at least an
unpleasant brush with Arabs in search of plunder. Probably they
were of the company at whose tents we met such kindness. The
man who will lay down his life to protect you when you have passed
under his roof may think you “fair game” if he finds you in the open.
This wady we passed early in the day, and thereafter we saw neither
stream, fountain, nor cistern through all the long burning hours.
Coming to what looked like an old burying-ground in the middle of
the plain, we halted for lunch, and waited for the muleteers. Two
hours passed before the “baggage train” appeared, from which we
judged that they had lost their way. Anxiety on their behalf could not
blind us to the almost certain futility of any search. Our decision to
wait in the somewhat conspicuous position we occupied and give
them a chance to find us was amply justified by the result. With
hands and face protected from the sun as best might be, we
stretched ourselves on the earth, and perhaps all indulged in the
luxury of “forty winks.” As time wore on, some sought entertainment
in trials of strength, agility, and skill—this last by shooting balls at a
stone set up a hundred yards away. The shot, even with a smooth
bore, was not alarmingly difficult; but when the stone toppled over,
the Arabs who were with us stood amazed. They might have done
as well themselves, but had not dreamed of Franjies shooting
straight. There is an impression abroad that Franjies are on the
whole rather helpless people, able perhaps to read and write. But
these are despicable attainments, save, indeed, when the former
may guide to the discovery of hid treasure!
Our baggage-men had wandered, much against their own will and
judgment, yielding to the head-strong ignorance of the man told off to
guide them. When his incompetence was plainly manifest, with
contemptuous anger they dismissed him, some bidding him hold by
his mother until he could walk alone, and others suggesting that
perhaps his wife needed him about the house. Then trusting their
own instinct, which, in these “sons of the highway” approaches
genius, they proceeded to find the road for themselves. The long
delay, however, destroyed all hope of seeing Jerash that night.
Soon after starting again we were joined by a bright, talkative youth,
who told us he was at work among the Beduw. He knew the various
tribes and their locations, and was familiar with most of the country.
He came from el-Judeideh, the summer station of the Sidon
American Mission, which overlooks Merj Aʿyûn, the ancient “Ijon,”
from the north-west. This village supplies many of the brave, light-
hearted fellows who drive their hardy beasts with the necessaries of
life through all parts of the land. Most mukaries (“muleteers”) have a
wholesome dread of the Lower Jordan Valley, but the men of el-
Judeideh may be seen there almost any day, swinging along with
careless ease, as much at home as on the slope of their native
mountain. This youth was accustomed to visit these parts every
spring, acting as Kâtib (“Secretary”) among the Bedawy chiefs, at the
time of division and arrangement of flocks. He assisted at bargains,
wrote out contracts, registered numbers, etc.; for these barbarous
sheyûkh, while holding it infra dig. for an Arab to write, quite realise
the value of “black and white” in a bargain. He busied himself for
some months, passing from tribe to tribe, and with advancing
summer turned again to his upland home. He assured us that Jerash
could not be reached that night, and urged us to turn aside with him
to a great Bedawy encampment, where we should find heartiest
welcome and plentiful entertainment. We lived to regret our refusal;
and, after all, we must have slept within an hour of the spot he
indicated—probably with a smaller branch of the same tribe. Our
guide vowed that water would be found on the edge of the plain, so
we judged it best to push on straight towards our goal. The lad bade
us gaily farewell, put spurs to his steed, and galloped away towards
a black patch at the base of the hills to westward, where no doubt
stood the Arabs’ “houses of hair.” A curl of smoke over an apparently
ruinous village away to the north-west was due, our guide said, to
the presence of Circassians, a number of whom had recently taken
possession and were attempting to cultivate the surrounding soil. If
any men could succeed, they should have a good chance. We shall
meet them again at Jerash.

PALESTINIAN SHEPHERD AND FLOCK


Amid a confused heap of hewn stones by the wayside we found a
broken column with a few fragments of Greek letters. It had served
as a milestone in ancient days, but could no longer yield information
as to the way. Nothing else arrested attention till we neared the edge
of the waste; then we were drawn to the left by a strip of green and
the music of many frogs—both indicating the presence of water.
Water we found, indeed, but so little of it and so vile, that not even
the thirsty animals would touch it. We came upon two huge
reservoirs, with never a drop in either. They stand one above the
other in the side of a gentle slope, carefully cemented, sides and
bottom, a flight of stone steps leading down into each. From certain
marks around, we thought they might have belonged to a system of
baths. On the hill above the reservoirs stands an old fortress, Kalʿat
Esdein. Still well preserved, all save the roof, which is gone, is a
building to the east, which may have been a church. A large cistern
within the court raised our hopes, only to dash them again. It was
quite empty. The fortress must have been of considerable strength,
built, as it is, round the top of a little hill, commanding the pass by
which we entered the country of ʿAjlûn.
We were tempted to halt here for the night, contenting ourselves with
dry fare, but the sight of our thirsty animals panting beside us, their
great eyes seeming to plead with us for water, moved our
compassions, so we set forward once more, although the sun was
already low in the west, and darkness comes without warning in
these lands. Some of us went in advance, hoping to find some wady
where little pools might still be left, or a spring under some green
wooded hill. Separating, we searched the country on either side of
the pass, taking what bearings were possible, so that we might not
lose the caravan, which wound along painfully below. Hill after hill
was scaled and valley after valley traversed, with ever the same
result. The shadows were falling thick when at last we struck a well-
beaten track, which we knew must lead to an Arab camp. The rest of
our party we saw on a hilltop behind us. The mukaries with the
baggage were a long way back. With no one to guide them, they
were sure to wander in the night that gathered over us, the darkness
deepened by great black clouds, that soon covered all the sky. As
the cook rode a powerful mare, it was hinted that he might return and
guide them past certain tempting openings. The poor man almost
shivered himself out of his saddle, a picture of abject terror. There
was nothing else for it, so I pushed forward my weary horse, marking
the hilltops against the sky. Some distance along the valley I heard
the music of the bells that hung tinkling round the necks of the
baggage animals, and guided by this, by and by came upon the
mukaries, moving cautiously for fear of ruts or holes in which the
mules might stumble. There was room enough for anxiety, but no
trace of it was seen in these fearless, happy-spirited children of the
mountain—no anxiety save what was excited by the condition of a
comrade who had fallen sick by the way. The kindness shown to the
sick youth, by these strong-limbed but tender-hearted men, was
most touching. They had an extra animal, which they rode by turns,
to rest their feet a little during the journey. This day and succeeding
days every man of them cheerfully gave up his “turn,” that their
fevered companion might ride all the way. It is hardly doing them
justice to say that they gave it up cheerfully: they never seemed to
think of it at all. Just as they came forward it was found that the sick
lad, in his weakness and weariness, had let something fall a good
way back. The big-hearted fellow who had been walking beside him
gave the others certain charges concerning him, and without even a
look of reproach, dived away into the shadows to search for it. Giving
the advancing party instructions as to the way, I stood to act as a
landmark, to guide the gallant Mousa on his return. The Bludân men,
reared in the bracing air of Anti-Libanus, are among the finest
specimens of the Syrian people. Independent, manly, yet withal
respectful, ever showing to advantage in difficulty or danger, their
tender solicitude for their unfortunate comrade did more to win our
hearts than all their more showy qualities.
Standing alone in the bottom of that thickly wooded vale, distant
objects already faded from sight, the hilltops themselves hardly
distinguishable against a sky that grew ever darker, flocks of vultures
fighting for places in the branches of trees near by, apparently
unused to fear in that solitude, I was not sorry to hear, at last, the
footfall of the returning Mousa. As we started forward together, a
bright flame leapt from the top of the highest hill before us. In the red
glare we could almost see the figures of our friends as they piled on
the fuel. The idea of the fire was excellent. The cook made the
suggestion, and fell to work with frantic energy, tearing up roots,
pulling down branches and heaping them up to burn, as if he hoped
the flame might scorch the reproach of cowardice from his accusing
conscience.
ARAB CAMP IN MT. GILEAD
Guided by the fire, we soon rejoined our companions on the hilltop.
The doctor, meantime, had found an Arab encampment, and
returned to lead us thither. His cheery voice rang out of the
darkness, calling us to follow him. It was only the voice we could
follow, as we never saw each other again until we gathered in the
ruddy light of the Bedawy fires. We came long after sunset,
committing thus unwillingly a breach of desert etiquette. But the
Arabs easily understood our plight, and soon great draughts of
delicious warm milk were provided. There is no better restorative
than this, after a fatiguing and anxious day. But our excitements
were not over yet. Great drops of rain slid down through the
darkness, as if the clouds perspired supporting their own weight.
Warning drops they were: we rushed up our tent before the shower
came which they heralded. Under its roof we all took refuge until the
mukaries had pitched a second tent; then we separated for the night,
to make the best of circumstances—sleeping on chairs, or stretched
on the canvas of our camp-beds, covered with anything that came to
hand.
Silence as of death had fallen over the mountains; not a leaf stirred
in the trees around us; sheep and oxen huddled closely together
beside the hair houses of their masters; and the clouds hung dark
and threatening, like birds of evil omen poised in the sky above us.
The darkness overhead was cloven as by a flaming scimitar, and out
rushed a stream of living fire, that spread for a moment over the hills
like a curtain of gleaming light, to which every particular leaf
responded with individual glitter. The thunder roared and bellowed
through all that empty land, like the mingling of the tornado with the
voice of many waters. The earth shook as if the very hills were being
hurled headlong in divine displeasure. The rain fell in torrents, and
beating on the taut canvas of our tents, served to increase the
uproar. It is impossible to exaggerate the grandeur of the scene. Not
till then could I fully appreciate the majestic realism of the famous old
song of the thunderstorm, Psalm xxix. Surely it was after witnessing
a storm like this that the Psalmist penned these marvellous
descriptive verses:
The voice of the Lord is upon the waters:
The God of glory thundereth,
Even the Lord upon many waters.
The voice of the Lord is powerful;
The voice of the Lord is full of majesty.
The voice of the Lord breaketh the cedars;
Yea, the voice of the Lord breaketh in pieces the cedars of
Lebanon.
He maketh them also to skip like a calf;
Lebanon and Sirion like a young wild-ox.
The voice of the Lord cleaveth the flames of fire.
The voice of the Lord shaketh the wilderness;
The Lord shaketh the wilderness of Kadesh.
The voice of the Lord maketh the hinds to calve,
And strippeth the forests bare.—Psalm xxix. 3-9.
As one reads, all seems to pass before him again in unparalleled
grandeur. In the midst of a scene like this, how completely one is
cast back upon the Lord Himself.
Once we could distinguish no interval between flash and crash, and
one of our company experienced a strange thrill passing through his
body. Mercifully we were preserved from serious injury. Sitting there
among the mountains, the worthy theatre of that awful display, the
poor Beduw near us crouching in abject fear beside their trembling
flocks, one could realise the comfort of the reflection with which the
poet concludes his song:
The Lord sitteth as King for ever.
The Lord will give strength unto His people;
The Lord will bless His people with peace.
CHAPTER X
Morning on the mountains—Arab time—Tents and
encampments—The Women and their work—Arab wealth
—Scenes at the wells—Dogs—Arabian hospitality—
Desert pests—Strange code of honour—The blood feud—
Judgment of the elders—Arab and horse—The Arabs and
religion—The Oriental mind—Arab visit to Damascus.
The storm continued all night, abating slightly towards midnight, but
increasing in violence as the fajr, or first glint of morning, stole into
the sky. A brilliant flash, followed instantly by a terrific crash, marked
the climax just as day broke. The dust, which had blown thick around
us on our approach, was transformed into soft, clinging mud. The
tents were so wet that packing was out of the question. The hoofs of
the animals sank deep into the yielding soil. Travel under these
conditions would be both slow and painful. We were fain to wait and
see what the sun would do for us. He soon rose in all his strength,
and in two hours worked wonders. As tents and roads grew dry, we
became more cheerful. The Arabs gathered in little groups,
submitting their ills to the doctor’s skill, giving what information they
could about our way, the country, and themselves. Jerash, we
learned, was only two or three hours distant. Not that the Arabs
knew anything of our method of measuring time. One said, “What do
we know about ‘hours’? But see: If the sun is there when you leave
here, it will be there when you reach Jerash”—pointing the two
quarters of the sky as he spoke.
The sun is the great time-keeper in the desert. By his pace all
journeys are measured. The three great points from which the Arabs
reckon are sunrise, noon, and sunset—Shurûk esh-shems, Zahr,
and Mughrab. When travelling, they like to be off before sunrise; and
no one is willingly abroad after sunset. They strive to reach some
friendly roof before the last beams of departing day have fled. It is
contrary to the etiquette of the Khâla (“empty waste”) for a guest to
throw himself upon his host after sunset. The evening meal, the chief
meal of the day, is eaten then. To arrive after it is prepared, or
finished, is to put the host to all the trouble of fresh preparation. This
no Arab would do if he could possibly avoid it. If the guest is of any
consideration, the host would grieve most of all that he was deprived
of the privilege of making a proper feast. Therefore, by common
consent it is said, “The guest who arrives after sunset goes
supperless to sleep.” There is, however, another reason for shunning
the shadows of early night. Then especially the robber pests of the
wilds ply their dark craft, with the long hours till morning in which to
flee. With such a start before his crime is discovered, and knowing
as he does the intricacies of the desert, the robber or murderer is
almost sure to escape.
The Arab “houses,” as they call them, are made of goats’-hair, spun
and woven by the women into long strips, about fifteen inches wide.
The weft is stretched on a frame; the woof is worked in by the
women’s fingers, and drawn up tightly with what looks like a huge,
short-toothed, wooden comb. Dark brown and white are the colours
mostly employed. These strips are sewn together with hair thread,
into pieces of sufficient breadth. Two poles are set up at each end of
the space to be covered. Over these the roof-cloth is stretched by
means of cords fastened to the ends, and attached to pins firmly
fixed in the ground. As many poles as are needed to support the roof
are introduced in the body of the “house,” and over these, by side
cords, tied as at the ends, to pegs in the ground, the cloth is drawn
taut. Often sufficient cloth is made for only one end and one side of
the “house.” This is fastened under the eaves, and is moved round
with the sun, so as to afford shade all day. This haircloth, once
thoroughly wet, draws so tightly together as to be perfectly
waterproof. Many think its rain-resisting qualities are improved by the
smoke of greenwood fires. It is the business of the women (el-harîm)
to put up the “house”; and among them it is reckoned a high
accomplishment to be able, with a single blow of the wooden mallet,
to drive the tent peg home. Jael the Kenite brought a practised hand
to drive the tent peg through the brow of the sleeping Sisera.
The tent is divided by a hair curtain drawn across the middle. One
end is the women’s, or more private family department, into which
strangers do not intrude. Here are kept the household stores, coffee,
rice, tobacco, samn, etc. Here also will generally be found the small
box, strongly bound with brass or iron, containing contracts, which
probably the owner cannot read, and any treasures to which more
than usual value is attached. A chief’s son on one occasion
produced, and displayed with no little pride, decorations which his
ancestors had received from European governments for services
rendered in troublous times. The other end of the tent is public,
where all gather on equal terms. Here the guest is received and
made to recline on cushions, which may be covered with silk if the
“master of the house” is a man of substance. A shallow hole at one
side is the “fireplace,” where coffee is prepared for the company.
Sometimes a large stone shields the fire from the wind.
The tents of an encampment are set end to end, with about the
space of a “house” between them. There may be but a single row, as
in the case of those with whom we had spent the night; but if the
number is large there may be two rows, forming a kind of street. The
place of honour is at the right hand as one enters the encampment;
and at either end this position is occupied by one of sheikhly rank.
The status of the householder may usually be inferred from the size
of his house; and this is reckoned by the number of poles necessary
to sustain it. The chief’s tent in the larger tribes provides
accommodation for many guests.

You might also like